Sei sulla pagina 1di 39

ANALOG IAS INSTITUTE TEST SERIES - 2016

C.S.(P)-2016

DO NOT OPEN THIS BOOKLET UNTIL YOU ARE ASKED TO DO SO


T.B.C. : P-PFC-L-CGJB
TEST BOOKLET SERIES
Serial No.

TEST BOOKLET

GENERAL STUDIES (PAPER I)


Time Allowed: Two Hours

INDIAN POLITY

Maximum Marks: 200

INSTRUCTIONS
1. IMMEDIATELY AFTER THE COMMENCEMENT OF THE EXAMINATION, YOU SHOULD CHECK THAT THIS
TEST BOOKLET DOES NOT HAVE ANY UNPRINTED OR TORN OR MISSING PAGES OR ITEMS, ETC. IF SO,
GET IT REPLACED BY A COMPLETE TEST BOOKLET.
2. ENCODE CLEARLY THE TEST BOOKLET SERIES A, B, C OR D AS THE CASE MAY BE IN THE
APPROPRIATE PLACE IN THE ANSWER SHEET.
3. You have to enter your Roll Number on the Test Booklet in the Box provided alongside. DO NOT write
anything else on the Test Booklet.
4. This test Booklet contains 100 items (questions). Each item is printed
both in Hindi and English. Each item comprises four responses
(answers). You will select the response which you want to mark on the
Answer Sheet. In case you feel that there is more than one correct
response, mark the response which you consider the best. In any case, choose ONLY ONE response for each
item.
5. You have to mark all your responses ONLY on the separate Answer Sheet provided. See directions in the
Answer Sheet.
6. All items carry equal marks.
7. Before you proceed to mark in the Answer Sheet the response to various items in the Test Booklet, you have
to fill in some particulars in the Answer Sheet as per instructions sent to you will your Admission Certificate.
8. After you have completed filling in all your responses on the Answer Sheet and the examination has
concluded, you should handover to the Invigilator only the Answer Sheet. You are permitted to take away
with you the Test Booklet.
9. USE ONLY BALL BLACK OR BLUE PEN TO MARK IN THE ANSWER SHEET.
10. Sheets for rough work are appended in the Test Booklet at the end.
11. Penalty for wrong answers:
THERE WILL BE PENALTY FOR WRONG ANSWERS MARKED BY A CANDIDATE IN THE OBJECTIVE TYPE
QUESTION PAPERS.
(i)There are four alternatives for the answer to every question. For each question for which a wrong answer
has been given by the candidate, one-third (0.333) of the marks assigned to that question will be deducted
as penalty.
(ii) If a candidate gives more than one answer, it will be treated as a wrong answer even if one of the given
answers happens to be correct and there will be same penalty as above to that question.
(iii) If a question is left blank, i.e., no answer is given by the candidate, there will be no penalty for that
question.

DO NOT OPEN THIS BOOKLET UNTIL YOU ARE ASKED TO DO SO

ANALOG IAS INSTITUTE


2nd Floor, 1-2-288/32, Indira Park 'X' Roads, Domalguda, Hyderabad 500029
Ph. No: 040-64590440, 9912441137
website: www.analogeducation.in

PRELIMS TEST SERIES 2016


INDIAN POLITY

Number of Questions: 100


1.

2.

3.

Marks: 200

Consider the following statements about


the Central Information Commission(CIC).
1. The CIC consists of a chairperson
and two other members.
2. MPs and MLAs are eligible to become
members of the CIC.
3. The chairperson should be an
eminent Justice in the opinion of the
President or eligible to be a Supreme
Court Judge.
Choose the correct answer from the codes
below
(a) 1 and 2 only
(b) 2 and 3 only
(c) 3 only
(d) None of the above
Which of the following are NOT eligible for
reappointment to the office?
1. Comptroller and Auditor General of
India.
2. Chief Information Commissioner.
3. Chairman, UPSC
4. Chief Election Commissioner.
Choose the correct answer from the codes
below
(a) 1, 3 and 4 only
(b) All of the above
(c) 1 and 2 only
(d) 2, 3 and 4 only
Consider the following statements about
the Constituent Assembly of India.
1. Its constitution was finally subject to
the
approval
of
the
British
Parliament.
2. The selection of representatives of
various Indian states was flexible
and decided by consultation.
3. Members of the CA from provinces
were elected directly from the people
by a very limited franchise.
Which of the following above is/are true?

4.

5.

6.

ANALOG IAS INSTITUTE The Right Choice of Achievers.


www.analogeducation.in

Time : 120 Min


(a) 1 and 2 only
(b) 2 and 3 only
(c) 1 and 3 only
(d) 2 only
Consider the following procedures:
1. President referring the matter to the
Lok Sabha.
2. Inquiry by the Supreme Court.
3. Parliament passes the motion by a
simple majority in both the houses.
Which of the following procedures is/are
followed for the removal of CVC?
(a) 2 and 3 only
(b) 1, 2 and 3 only
(c) 2 only
(d) 1 and 3 only
Which of the following domains are
handled by the CBI for investigation and
reporting?
1.
Cyber and high technology crime
2.
Transnational organized crimes
3.
Economic crimes
Choose the correct answer from the codes
below
(a) 1 and 2 only
(b) 2 and 3 only
(c) All of the above
(d) 3 only
The Government of India Act, 1919
popularly
known
as
Montague
Chelmsford Act was significant in which
of the following ways?
1. Established
a
responsible
Parliamentary system of Government
in the country.
2. The final decision on all important
questions was no more in the hands
of the Viceroy.
3. It
relaxed
previously
stringent
control of center on provinces.
Which of the following above is/are true?
Page |2
ias.analog@gmail.com

(a) 1 and 2 only


(b) 2 and 3 only
(c) 1 and 3 only
(d) 3 only
7.
The 97th Constitution Amendment Act of
2011 gave a constitutional status and
protection to co-operative societies. In this
context, which of the following changes
were made in the Constitution?
1. Right to form co-operative societies
became Fundamental Right.
2. It includes a new Directive Principle
of State Policy on promotion of cooperative societies.
3. It established a central constitution
authority for overseeing matters
related to cooperatives.
Choose the correct answer from the codes
below:
(a) 1 and 2 only
(b) 2 and 3 only
(c) All of the above
(d) 3 only
8.
Consider
the
following
about
the
composition, powers and appointments
related to the Central Administrative
Tribunal(CAT).
1. The Chairman and members are
appointed by a collegiums consisting
of senior Supreme Court judges
headed by the Chief Justice of India.
2. The members are drawn from both
judicial and administrative stream.
3. CAT orders and decrees are final,
and no provision of appeal lies
whatsoever.
Choose the correct answer from the codes
below
(a) 1 and 2 only
(b) 2 and 3 only
(c) 2 only
(d) 1 and 3 only
9.
Consider the following statements about
Central Services and All-India Services.
1. Any disciplinary action (imposition of
penalties) against these officers can
only be taken by the central
government.
2. The personnel of Central services
(even though are posted in states)

10.

11.

12.

13.

ANALOG IAS INSTITUTE The Right Choice of Achievers.


www.analogeducation.in

work under the exclusive jurisdiction


of the central government.
Which of the following above is/are true?
(a) 1 only
(b) 2 only
(c) Both 1 and 2
(d) None
The list of castes or tribes that are to be
identification as SCs and STs are
specified in
(a) A presidential order with respect to
each State and Union Territory(UT).
(b) The Fifth and Sixth Schedule of the
Constitution of India
(c) The resolution passed by the
respective state legislatures.
(d) The orders issued by the state
Governor in the extra-ordinary
Gazette of India
The Election Commissioner registers
political parties for the purpose of
elections and grants them recognition as
national or state parties on the basis of
their
1. Assets and liabilities
2. Geographical presence in more than
four constituencies.
3. Number of workers, volunteers and
election candidates.
4. Poll performance.
Choose the correct answer from the codes
below
(a) 1 and 4 only
(b) 2 and 4 only
(c) 1 and 3 only
(d) 4 only
Disqualification on ground of defection
does not apply in cases of
1. Political party mergers
2. Political party splits
3. A candidate quitting political party
Choose the correct answer from the codes
below
(a) 1 and 2 only
(b) 2 and 3 only
(c) 1 and 3 only
(d) 2 only
In the table of precedence, which of the
following/authorities
precedence
the
Chief Election Commissioner?
Page |3
ias.analog@gmail.com

14.

15.

16.

1. Holders of Bharat Ratna decoration


2. Judges of Supreme Court
3. Attorney General of India
4.
Leader of Opposition, Loksabha
Choose the correct answer from the codes
below:
(a) 1 and 2 only
(b) 1, 2 and 4 only
(c) 2 and 3 only
(d) 1 and 4 only
Consider the following statements.
1. There is only one general electoral
roll for every territorial constituency
for elections to the Parliament and
the State Legislature.
2. The State Legislatures, apart from
Parliament can also make provision
with respect to all matters relating to
the elections to the State Legislature.
Which of the above is/are true?
(a) 1 only
(b) 2 only
(c) Both 1 and 2
(d) None
Which of the following features of the
Indian Constitution deal with maintaining
the social fabric, moral values and
national consciousness?
1. Fundamental Rights
2. Fundamental Duties
3. Directive Principles of State Policy
Choose the correct answer from the codes
below?
(a) 1 and 2 only
(b) 2 and 3 only
(c) 1 and 3 only
(d) All of the above
Which of the following category of
Constitution amendments would require
the ratification of the states in India?
1. Citizenship clauses
2. Changes in the Concurrent List
under Seventh Schedule.
3. Changes in the State boundaries
4. Changing the number of puisne
judges in the Supreme Court
Choose the correct answer from the codes
below?
(a) 1 and 4 only
(b) 2 and 3 only

17.

18.

19.

ANALOG IAS INSTITUTE The Right Choice of Achievers.


www.analogeducation.in

(c) 1, 2 and 3 only


(d) 2 only
Which of the following possibilities can be
found in Parliamentary Democracy?
1. It can be a Constitutional monarchy
2. President can be head of the
government
3. The Prime Minister may belong to
the party without majority in the
legislature.
Choose the correct answer from the codes
below?
(a) 1 and 2 only
(b) 2 and 3 only
(c) 1 and 3 only
(d) All of the above
Consider the following statements about
the position, power, functions and
responsibilities of the President of India.
1. He has a right to be informed of all
important matters and deliberations
of the Council of Ministers.
2. The president need not accept the
advice of Council of Ministers if the
advice has been given for the first
time.
3. The
only
ground
for
his
impeachment is the violation of the
Constitution.
4. He scrutinizes the annual budget
proposal of the government before
placing it to the Parliament.
Choose the correct answer from the codes
below?
(a) 1 and 2 only
(b) 1, 2 and 3 only
(c) 2 and 3 only
(d) All of the above
Consider a situation where the Prime
Minister wants to impose Presidents rule
in one state because the state government
has failed to effectively curb atrocities in
that state. The president does not agree
wholeheartedly on signing the order.
Which of the following course of actions
are available to the President?
1. Tell the Prime Minister that he will
not sign on the order promulgating
Presidents rule.

Page |4
ias.analog@gmail.com

2.

20.

21.

22.

Make press statement about how the


Prime Minister is wrong.
3. Discuss the matter with the Prime
Minister and try to dissuade him
from taking this action, but if he
insists agreeing to sign the said
order.
Choose the correct answer from the codes
below
(a) 1 only
(b) 2 and 3 only
(c) 1 and 3 only
(d) 3 only
Consider the following about the advisory
jurisdiction of the Supreme Court (SC)
under article 143 of the Constitution.
1. Being a constitution organ, the SC is
bound to give advice when it is
referred.
2. The SC being the highest court of
Justice, legal advice tendered by the
court is legally binding on the
President.
Which of the following above is/are true?
(a) 1 only
(b) 2 only
(c) Both 1 and 2
(d) None
Which of the following articles of the
Indian
Constitution
provides
the
safeguard against the excess of the
legislature and the executive?
1. Article 13
2. Article 19
3. Article 21
4. Article 32
Choose the correct answer from the codes
below
(a) 1 and 4 only
(b) 2 and 4 only
(c) 1, 2 and 3 only
(d) All of the above
Consider
the
following
about
the
proportional representation system of
election.
1. The entire country may be single
constituency in this system.
2. Multi member constituencies may be
present.

3.

23.

24.

25.

ANALOG IAS INSTITUTE The Right Choice of Achievers.


www.analogeducation.in

A party may get more seats than


votes in the legislature.
Choose the correct answer from the codes
below
(a) 1 and 2 only
(b) 2 and 3 only
(c) 1 and 3 only
(d) All of the above
Consider the following statements with
reference to the status of J&K in the
Indian federal structure.
1. The central legislation in the Union
list and Concurrent list applies to
J&K only after the consent of the
state government.
2. The Directive Principles of state
policy do not apply to the state.
3. Amendments
to
the
Indian
Constitution (under Art.368) can
only apply in concurrence with the
government of J&K.
Choose the correct answer from the codes
below
(a) 1 and 2 only
(b) 2 and 3 only
(c) 1 and 3 only
(d) All of the above
In the state government, the Governor
exercise more discretion than the
President can exercise in the case of
Central Government. This is because.
(a) Indian Federalism provides for a
strong central government, and
comparatively weaker states.
(b) The Constitutional provisions limit
the
overall
discretion
of
the
President, which is not so clearly
defined for the Governor.
(c) The
Governor
derives
greater
authority from the order of the
Central government with respect to
the states whereas no such system
exists for the President.
(d) The state politics is much more
vulnerable to Governors discretion
that the central politics is to
Presidents discretion.
The Constitution contains not only the
fundamental principles of governance but
also detailed administrative provisions.
Page |5
ias.analog@gmail.com

26.

27.

28.

This fact can be supported by looking up


in which of the following provisions of
Constitution?
1. Fifth and Sixth Schedule
2. Center-State relations
3. Appointment
and
functions
of
several constitution bodies.
Choose the correct answer from the codes
below?
(a) 1 and 2 only
(b) 2 and 3 only
(c) 1 and 3 only
(d) All of the above
Which of the following do NOT require a
Constitution amendment?
1. Settlement of boundary dispute
between India and other country.
2. Changing the name of a state
3. Creation of new state
Choose the correct answer from the codes
below?
(a) 1 and 2 only
(b) 2 and 3 only
(c) 1 and 3 only
(d) All of the above
Consider the following statements:
1. In India, citizens by birth as well as
a naturalized citizen are eligible for
the office of the President.
2. Foreigners staying in India do not
have to oblige to the Fundamental
Duties.
3. The Constitution does not prescribe
any qualification for citizenship.
Choose the correct answer from the codes
below?
(a) 1 and 2 only
(b) 2 and 3 only
(c) 1 and 3 only
(d) All of the above
Consider the following statements about
the Constituent Assembly (CA).
1. It was based on scheme suggested
by the cabinet mission.
2. All provinces and princely states
were to be represented in the CA as
per decided scheme.
3. There were to be nominated
members in the CA as per decided
scheme.

29.

30.

31.

ANALOG IAS INSTITUTE The Right Choice of Achievers.


www.analogeducation.in

Choose the correct answer from the codes


below
(a) 1 and 2 only
(b) 2 and 3 only
(c) 1 and 3 only
(d) All of the above
The structural part of the Constitution is,
to a large extent, derived from the
Government of Indian Act 1935. This
argument is supported by which of the
following features/provisions of the
Constitution?
1. Public Service Commission of Union
and States
2. Federal polity
3. Division of powers in lists between
Center and States
4. Bicameralism in state assemblies.
Choose the correct answer from the codes
below?
(a) 1 and 2 only
(b) 2 and 3 only
(c) 1 and 4 only
(d) All of the above
India has adopted the Parliamentary
system of Government. It is based on the
principle of
(a) Doctrine of separation of powers
between
the
legislature
and
executive organs
(b) Cooperation
and
Co-ordination
between the legislative and executive
organs
(c) Doctrine of separation of powers
between the legislature and judicial
organs
(d) Doctrine of separation of powers
between the judicial and executive
organs
Consider the following statements
1. As per the Independence Act of
1947, the Constitution Assembly
was to become the first domain
legislature.
2. The Constitution Assembly was not a
fully sovereign body.
Which of the following above is/are true?
(a) 1 only
(b) 2 only
(c) Both 1 and 2
Page |6
ias.analog@gmail.com

32.

33.

34.

35.

(d) None
Among the committees of the Constitution
Assembly, the most important committee
was the Drafting Committee. Who among
the following were NOT the members of
the committee?
1. N.GopalaswamyAyyangar
2. Syed Mohammad Saadullah
3. T.T. Krishnamachari
4. Dr.K.M.Munshi
Choose the correct answer from the codes
below
(a) 4 only
(b) 2 and 3 only
(c) 1 only
(d) All were the members
In India, all citizens irrespective of the
state in which they are born or reside,
enjoy the same political and civil rights of
citizenship all over the country and no
discrimination is made between them
excepting in few cases in
1. Tribal areas
2. State of J&K
3. Areas mentioned under Article 371
of the Constitution
Choose the correct answer from the codes
below
(a) 1 and 2 only
(b) 2 and 3 only
(c) 1 and 3 only
(d) All of the above
State Emergency or Presidents Rule can
be applied to an Indian state on which of
the following grounds?
1. Failure to comply with Centers
directions
2. Breakdown of the Constitutional
machinery in the state
3. Threat to the financial stability of the
state
4. Internal aggression in the state
Choose the correct answer from the codes
below
(a) 1 and 2 only
(b) 2 and 4 only
(c) 1, 3 and 4 only
(d) 1, 2 and 3only
Consider the following statements about
the Preamble of the Constitution.

1.
2.

36.

37.

38.

ANALOG IAS INSTITUTE The Right Choice of Achievers.


www.analogeducation.in

The Preamble is not justiciable.


It declares the nature of the Indian
state.
3. It envisages political as well as social
ideals.
Choose the correct answer from the codes
below
(a) 1 and 2 only
(b) 2 and 3 only
(c) 1 and 3 only
(d) All of the above
Consider the following statements
1. Territory of India is a wider
expression than the Union of India
as the former includes only states &
UTs.
2. India can acquire foreign territories
according to models recognized by
international law.
3. Parliament can establish new states
that were not a part of the Union of
India.
Choose the correct answer from the codes
below
(a) 1 and 2 only
(b) 2 and 3 only
(c) 1 and 3 only
(d) All of the above
The term State has been used in
different
provisions
concerning
the
Fundamental Rights against which the
citizens are protected. State can include
which of the following?
1. A public sector undertaking
2. District Planning Committee
3. A Private party under a Public
Private Partnership(PPP)
contract
with the government
Choose the correct answer from the codes
below
(a) 1 and 2 only
(b) 2 and 3 only
(c) 1 and 3 only
(d) All of the above
As per the Supreme Court which of the
following included in the Freedom of
Speech and Expression?
1. Right against tapping of telephonic
conversation

Page |7
ias.analog@gmail.com

2.

39.

40.

41.

Right to know about government


activities
3. Right against bandh called by a
political party or organization
Choose the correct answer from the codes
below
(a) All of the above
(b) 2 and 3 only
(c) 1 and 3 only
(d) 1 and 2 only
Article 23 of the Constitution prohibits
traffic in human beings, forced labour
and other similar forms of forced labour.
It is enforced by which of the following
laws made by the parliament?
1. Minimum wages Act, 1948
2. Contract labour Act, 1970
3. Equal Remuneration Act, 1976
Choose the correct answer from the codes
below
(a) All of the above
(b) 2 and 3 only
(c) 1 and 3 only
(d) 1 and 2 only
Although India is a secular country, the
state can take some steps to ensure
public order, morality and the like. Which
of the following can be done by the state
constitutionally?
1. Regulate a religious institution
2. Throw
open
Hindu
religious
institutions of a public character to
all classes and section of Hindus.
3. Restrict a religious practice
Choose the correct answer from the codes
below?
(a) 1 and 2 only
(b) 2 and 3 only
(c) 1 and 3 only
(d) All of the above
In what respects the writ jurisdiction of
the Supreme Court is narrower and less
discretionary than that of a High Court?
1. High courts can enforce legal rights
too whereas SC cannot
2. High court can issue it both against
persons and state, SC can issue it
only against the later.

3.

42.

43.

44.

ANALOG IAS INSTITUTE The Right Choice of Achievers.


www.analogeducation.in

SC cannot refuse to issue writs when


it comes to Fundamental Rights
whereas High courts can.
Choose the correct answer from the codes
below?
(a)1 and 2 only
(b) 2 and 3 only
(c) 1 and 3 only
(d) All of the above
Consider the following statements about
the Right to Freedom from taxation for
promotion of religion.
1. The state cannot use public tax
money for the promotion of any
religion.
2. The state cannot impose a fee on any
religious activities.
Which of the following above is/are true?
(a) 1 only
(b) 2 only
(c) Both 1 and 2
(d) None
Judicial Review in India is based on the
procedure established by law contained
in the Indian Constitution (Article 21).
What is meant by this principle?
(a) Courts can review a parliamentary
enactment if it is not reasonable.
(b) Courts can review a parliamentary
enactment if it goes against the goals
and objectives enshrined in the
Constitution.
(c) Courts can review a Parliamentary
enactment only if it was not enacted
following the right procedure.
(d) None of the above
The Directive Principles of the State
Policy denote the ideals that the state
should keep in mind while formulating
policies and enacting laws. These
principles are applicable to which of the
following organs?
1. Legislative
2. Executive
3. Local authority
4. Public sector enterprises(PSUs)
Choose the correct answer from the codes
below
(a) All of the above
(b) 1, 2 and 3 only
Page |8
ias.analog@gmail.com

45.

46.

47.

48.

(c) 1 only
d) 2 and 3 only
In the colonial legacy of government in
India, the Directive Principles in the
present Indian Constitution resemble
which
of
the
following
colonial
enactments?
(a) Codes enacted under Act of 1919
(b) Instrument of Instructions enacted
under Independence Act, 1947
(c) Instrument
of
Instructions
enumerated in the Government of
India Act of 1935
(d) Duties of State enumerated in the
Morley-Minto Act of 1909
The 97th Amendment act of 2011 added
new Directive Principles relating to cooperative societies in the Constitution.
The amendment is concerned with which
of the following aspects of cooperative
societies?
1. Autonomous functioning
2. Democratic control
3. Professional management
Choose the correct answer from the codes
below
(a) 1 and 2 only
(b) 2 and 3 only
(c) 1 and 3 only
(d) All of the above
For a citizen of India, the duty to pay
taxes is a:
(a) Fundamental Duty
(b) Legal Obligation
(c) Constitutional Obligation
(d) Moral Obligation
Which of the following is a Fundamental
Duty mentioned in Article 51-A of the
Constitution?
1. To develop a sprit of inquiry
2. To respect National Flag and
National Anthem
3. To safeguard public property
4. To do charitable activities which
promotes social and economic equity
Choose the correct answer from the codes
below
(a) All of the above
(b) 1, 2 and 3 only
(c) 1 and 2 only

49.

50.

51.

52.

ANALOG IAS INSTITUTE The Right Choice of Achievers.


www.analogeducation.in

(d) 3 and 4 only


As per Article 368 of the Constitution, the
amendment of the Indian Constitution
can be initiated in
1. Lok Sabha
2. Rajya Sabha
3. The office of the President
4. State Legislatures
5. Council of Ministers
Choose the correct answer from the codes
below
(a) 1, 3 and 5 only
(b) 1 and 2 only
(c) 2, 3 and 4 only
(d) 1, 2 and 5 only
A
number
of
provisions
in
the
Constitution can be amended by a simple
majority of the two Houses of Parliament
outside the scope of Article 368. These
provisions include, inter alia
1. Citizenship clauses
2. Abolition or creation of legislative
councils in states
3. Election of the President and its
manner
Choose the correct answer from the codes
below
(a) 1 and 2 only
(b) 2 only
(c) 1 and 3 only
(d) None of the above
Collective responsibility of the Council of
Ministers to the Parliament is the bedrock
principle of parliamentary government. It
implies that
(a) The parliament can remove the
Council of Ministers from office by
passing a vote of no confidence.
(b) The
Council
of
Ministers
is
appointed and dismissed by the
Parliament
(c) All important decisions of the
Council of Ministers have to be
approved by the Parliament
(d) None of the above
In which of the following cases the Lok
Sabha may be dissolved?
1. Prime Minister recommending the
dissolution of Lok Sabha to the
President
Page |9
ias.analog@gmail.com

2.

53.

54.

55.

No-confidence motion passed against


the incumbent government
3. Resignation of majority of cabinet
ministers
Choose the correct answer from the codes
below
(a) 1 and 2 only
(b) 2 and 3 only
(c) 1 and 3 only
(d) All of the above
Consider the following statements
1. The design of the national flag was
adopted
by
the
Constitution
Assembly of India became republic.
2. The display of the National Flag is
solely governed by the non-statutory
instructions
issued
by
the
government time to time
Which of the following above is/are true?
(a) 1 only
(b) 2 only
(c) Both 1 and 2
(d) None
Consider the following statements
1. The President of India is the
Constitutional head of executive of
the Union.
2. The Council of Ministers are
collectively responsible to both the
houses of the Parliament.
Which of the following above is/are true?
(a) 1 only
(b) 2 only
(c) Both 1 and 2
(d) None
Consider the following statements
1. National emergency can be declared
even if security of India is not in
threat, but there is a case of
imminent danger.
2. The operation of the National
emergency always applies to the
whole of India Territory.
Which of the following above is/are true?
(a) 1 only
(b) 2 only
(c) Both 1 and 2
(d) None

56.

57.

58.

59.

ANALOG IAS INSTITUTE The Right Choice of Achievers.


www.analogeducation.in

Constitution provisions which have a


bearing on planning in the country can be
found in
1. Seventh Schedule
2. Part IX
3. Sixth Schedule
Choose the correct answer from the codes
below
(a) 1 and 2 only
(b) 2 and 3 only
(c) 1 and 3 only
(d) All of the above
With reference to the office of Vice
President of India, consider the following
statements:
1. When he is acting as president, he
still performs the function of the
Chairman of the Rajya Sabha
2. The
Vice-President
is
elected
indirectly by members of an electoral
college consisting of the members of
both Houses of Parliament.
Which of the following above is/are true?
(a) 1 only
(b) 2 only
(c) Both 1 and 2
(d) None
With reference to the Rajya Sabha,
consider the following statements
1. It is not subjected to dissolution
2. One-third of its members retire every
second year
3. All the elected members of the Rajya
Sabha are indirectly elected
Which of the following statements is/are
Incorrect?
(a) 1 only
(b) 2 only
(c) 3 only
(d) None
In case of a conflict between the central
law and the state law on a subject
enumerated in the Concurrent List, which
of the following is possible?
1. Central law prevails over the State
law
2. State law prevails if it has received
Presidential assent

P a g e | 10
ias.analog@gmail.com

3.

60.

61.

62.

State law prevails if Governors of two


or more states have approved the
same legislation
Choose the correct answer from the codes
below
(a) 1 and 2 only
(b) 2 and 3 only
(c) 1 and 3 only
(d) All of the above
The
Constitution
empowers
the
Parliament to make laws on any matter
enumerated in the state list under which
of
the
following
extraordinary
circumstances?
1. When states make request
2. To
implement
International
agreements
3. When Rajya Sabha passes a
resolution to that effect
4. Presidents
order
in
the
extraordinary gazette of India
Choose the correct answer from the codes
below
(a) All of the above
(b) 1, 2 and 3 only
(c) 1, 2 and 4 only
(d) 3 and 4 only
The Constitution provides for a division of
taxation powers between Center and
States. Among several taxes, Service tax
is
(a) Levied by the Center but Collected
and Appropriated by the States
(b) Levied by the Center but Collected
and Appropriated by the States and
Center
(c) Levied, collected and retained by the
Center
(d) Levied, collected and retained by the
states
Article 262 of the Constitution provides
for the adjudication of inter-state water
disputes by a separate tribunal. The need
for an extra judicial machinery to settle
inter-state water disputes is because
(a) The court being overburdened with
litigation and adjudication slowly are
incapable of adjudicating sensitive
water disputes

(b)

63.

64.

65.

ANALOG IAS INSTITUTE The Right Choice of Achievers.


www.analogeducation.in

Water resources are not private


property and so rule of law is applied
by ordinary courts which is not
appropriate
to
deal
with
its
distribution
(c) Division and distribution of any
natural resources is out of judicial
scrutiny in India
(d) The courts cannot employ technical
committee to ascertain in the
distribution of water resources.
With reference to Comptroller and
Auditory General of India, consider the
following statements
1. The reports of Comptroller and
Auditory General of India relating to
the accounts of the states shall be
submitted to the President,
who
shall cause them to be laid before
concerned legislative assemblies
2. He shall only be removed from office
in like manner and no like grounds
as a judge of the Supreme Court
Which of the following above is/are true?
(a) 1 only
(b) 2 only
(c) Both 1 and 2
(d) None
Consider the following statements
1. The formation of tribunals in India
owes their origin to Constitutional
provisions
2. The administrative tribunals exercise
original jurisdiction in respect of
service matters of employees covered
by them
Which of the following above is/are true?
(a) 1 only
(b) 2 only
(c) Both 1 and 2
(d) None
With reference to electoral rolls, consider
the following statements
1. The electoral roll is a list of all people
in the constituency who are
registered to vote in Indian Elections
2. Only those people with their names
on the electoral roll are allowed to
vote

P a g e | 11
ias.analog@gmail.com

3.

66.

67.

68.

The electoral roll is revised once


every five years
Choose the correct answer from the codes
below
(a) 1 and 3 only
(b) 1 and 2 only
(c) 2 and 3 only
(d) All of the above
The Parliament exercise controls over the
Government using which of the following
devices?
1. Question Hour
2. Adjournment motion
3. Discussion
4. Parliamentary committees
Choose the correct answer from the codes
below
(a) All of the above
(b) 1, 2 and 3 only
(c) 1 and 2 only
(d) 3 and 4 only
Consider
the
following
statements
regarding the powers of both the houses
of a state legislature, if it is bicameral, in
India:
1. An ordinary bill can be introduced in
any house of the legislature.
2. A joint sitting of both the houses
may be summoned if there is a
disagreement between both the
houses on an ordinary bill
3. The Legislative Council has almost
no powers in relation to finance
Which of the following above is/are true?
(a) 1 and 2 only
(b) 2 and 3 only
(c) 1 and 3 only
(d) 1, 2 and 3
If any question arises as to whether a
member of either house of the Parliament
is subjected to disqualification, under
Article 102 (2) of the Indian Constitution,
to whom shall such question be referred
in order to make a final decision?
(a) President of India
(b) Election Commission of India
(c) Speaker of the Lok Sabha or
Chairperson of the Rajya Sabha
(d) Chief Justice of India

69.

70.

71.

ANALOG IAS INSTITUTE The Right Choice of Achievers.


www.analogeducation.in

Consider
the
following
statements
regarding the committees of the Indian
Parliament:
1. The members of the Public Accounts
Committee are drawn entirely from
the Lok Sabha.
2. The Estimates Committee scrutinize
reports of the Comptroller and
Auditor General
3. The Public Accounts Committee
suggested alternative policies in
order to bring about efficiency and
economy in administration.
Which of the following statements given
above is/are correct?
(a) 1 only
(b) 2 and 3 only
(c) 3 only
(d) None of the above
Which among the following statements
is/are correct regarding the Indian
Constitution?
1. To amend the Constitution under
Article 368, two third majority of the
total membership of the houses of
the Parliament is required.
2. If any question arises as to whether
a Bill is a Money Bill or not, the
decision of the President shall be
final
3. No person shall be prosecuted and
punished for the same offence more
than
once,
including
any
departmental proceedings
Select the correct answers from the codes
given below:
(a) 1 and 2 only
(b) 2 and 3 only
(c) 1 and 3 only
(d) None of the above
Who among the following are parts of the
Union Executive of India?
1. The President
2. Council of Ministers
3. Attorney General
4. Comptroller and Auditor General
Select the correct answers from the codes
given below:
(a) 1, 3 and 4 only
(b) 2, 3 and 4 only
P a g e | 12
ias.analog@gmail.com

72.

73.

74.

75.

(c) 1, 2 and 3 only


(d) 1, 2 and 4 only
The national Capital Territory of Delhi
was formed under 69th Constitutional
Amendment Act, 1991. Under this Act
which of the following subjects mentioned
in the List-2 of the Seventh Schedule in
the
Indian
Constitution
was/were
specifically excluded from the jurisdiction
of the Government of NCT of Delhi
1. Public Order
2. Police
3. Land
4. Taxes
Select the correct answers from the codes
given below:
(a) 1 and 2 only
(b) 1, 2 and 3 only
(c) 2 only
(d) 1, 2, 3 and 4 only
According to Preamble of the Indian
Constitution, India is a Republic. What is
the meaning of the Republic?
1. There is no hereditary rule
2. The sovereignty resides in the people
3. The head of the Government is
elected by the people of fixed term
Select the correct answers from the codes
given below:
(a) 1 and 2 only
(b) 2 and 3 only
(c) 1 and 3 only
(d) 1, 2 and 3
Consider the following Rights:
1. Right to equality before law
2. Right to Freedom
3. Right to Protection of Life and
Personal Liberty
The Indian Constitution provides which of
the above Fundamental right/rights only
to its Citizens?
(a) 1 only
(b) 2 only
(c) 2 and 3 only
(d) 1 and 3 only
Consider the following Statements:
1. The concept of Public Interest
Litigation (PIL) is in consonance with
the principles enshrined in Article
39A of the Constitution of India

76.

77.

ANALOG IAS INSTITUTE The Right Choice of Achievers.


www.analogeducation.in

which talk about the need to protect


and deliver prompt social justice
with help of law.
2. Justice PN Bhagwati and Justice VR
Krishna Iyer were among the first
judges to admit PILs in court.
3. A PIL may be introduced in a court
of law by the court itself (suo motu),
rather than by the aggrieved party or
by a third party.
4. In a PIL the right to file a petition is
given to a member of the Bar Council
of India by the courts through
judicial activism.
Which of the following statements given
above is/are correct?
(a) 1, 2 and 3 only
(b) 1, 3 and 4 only
(c) 2, 3 and 4 only
(d) All of the above
Which of the following statements
regarding Indian National Commission for
Scheduled Castes is/are correct?
1. The commission presents its annual
report to the Prime Minister
2. The Prime Minister places the
reports of the Commission before the
Parliament.
3. The Central government and state
government are required to consult
the commission on all major policy
matters affecting the schedule
castes.
Select the correct answer using the codes
below:
(a) 1 and 2 only
(b) 2 and 3 only
(c) 3 only
(d) none of these.
Which
of
the
following
is
the
constitutional duty of the Prime Minister
of India?
(a) To communicate to the President all
decisions of the Council of Ministers
relating to the administration of the
affairs of the union and to proposals
for legislation.
(b) To take prior permission of the
President for the introduction of bills
in the Parliament.
P a g e | 13
ias.analog@gmail.com

78.

79.

80.

To present all Parliament reports as


well as recommendations of various
commissions to the President
(d) To take the Presidents permission
for the appointment of the Protem
speaker of the Lok Sabha
Which of the following statements
regarding the special status of Jammu
and Kashmir is/are correct?
1. The Fundamental Right to property
is still guaranteed in Jammu and
Kashmir.
2. The President of India has no power
to declare a financial emergency in
relation to the state.
3. An amendment made to the
Constitution of India does not apply
to the state unless it is extended by
a Presidential order.
Select the correct answer using the codes
below:
(a) 1 and 2 only
(b) 2 and 3 only
(c) 3 only
(d) 1, 2 and 3
Which of the following statements
regarding voting on demand of grants
is/are correct ?
1. The voting on demands for grants is
an exclusive privilege of the Rajya
Sabha; Lok Sabha has no power on
demands.
2. The voting confined to the votable
part of the budget; the expenditure
charged on the Consolidated Fund of
India is not submitted to vote
Select the correct answer using the codes
below:
(a) 1 only
(b) 2 only
(c) Both 1 and 2
(d) Neither 1 nor 2
Consider
the
following
Statements
regarding the Chief Minister of a state in
India:
1. The salary and allowance of the
Chief Minister are determined by the
President of India on the advice of
the governor of concerned state.

2.

(c)

81.

82.

ANALOG IAS INSTITUTE The Right Choice of Achievers.


www.analogeducation.in

The Chief Minister acts as the


chairman of the concerned zonal
council by rotation, holding office for
a period of Three years at a time
Select the correct answer using the codes
below:
(a) 1 only
(b) 2 only
(c) Both 1 and 2
(d) Neither 1 nor 2
Which of the following statements
regarding conditions for recognition of a
political party as a national party (in
India) is/ are correct?
1. The party should have owned at
least 2% of seats in the Lok Sabha
(11 seats) from three different states.
2. In a general election to Lok Sabha or
Legislative Assemblies, the party
should have got at least 8% of polled
votes in six different states and, in
addition, it should have won six Lok
Sabha seats.
3. The
party
should
have
got
recognition as the state party in six
or more states.
Select the correct answer using the codes
below:
(a) 1 only
(b) 2 only
(c) 2 and 3 only
(d) 1, 2 and 3
Consider the following Statements about
the speaker of a legislative assembly of a
state in India:
1. He decides whether a bill is a Money
bill and his decision on it is final
2. He is the chairmen of the business
Advisory
Committee,
Rules
Committee and General Purpose
Committee of the concerned state.
3. A resolution passed by a majority of
two-third of all the then members of
the house is required for his removal
from his office.
Select the correct answer using the codes
below:
(a) 1 and 2 only
(b) 2 and 3 only
(c) 1 and 3 only
P a g e | 14
ias.analog@gmail.com

83.

84.

85.

(d) 1, 2 and 3
Consider
the
following
Statements
regarding the administration of Schedule
and tribal areas in India:
1. Each state having schedule areas
has to establish a Tribes advisory
council.
2. Three-fourth of the members of
advisory council of the concerned
state are to be representatives of
scheduled tribes in the state
legislative assembly.
3. If there are different tribes in an
autonomous district, the Governor of
the concerned state can divide the
district into several autonomous
regions.
Select the correct answer using the codes
below:
(a) 1 and 2 only
(b) 2 and 3 only
(c) 1 and 3 only
(d) 1, 2 and 3
Which of the following are components of
Inter-State Council?
1. Prime Minister
2. Chief Minister of all States
3. Chief Ministers of Union Territories
that have a legislative Assembly and
Administrative of UTs that do not
have a Legislative Assembly
4. Six Union Ministers
5. Governor if a state is under
Presidents rule.
Select the correct answer using the codes
below:
(a) 1 and 2 only
(b) 1, 2, 3 and 4 only
(c) 1, 2 and 4 only
(d) 1, 2, 3, 4 and 5
Consider the following statements:
1. The allocation of seats in Rajya
Sabha to the states is based on
population of that particular state.
2. A member of a Rajya Sabha should
be a domicile of the state from which
he/she is elected
3. Members of the Rajya Sabha are
elected by the elected members of

86.

87.

88.

ANALOG IAS INSTITUTE The Right Choice of Achievers.


www.analogeducation.in

the Assemblies of States and Union


Territories.
Which of the following statements given
above is/are correct?
(a) 1 and 2 only
(b) 2 only
(c) 1 and 3 only
(d) 1, 2 and 3
Which of the following features of the
Constitution of India have been borrowed
from the Government of India Act, 1935?
1. Office of the Governor
2. Emergency Provisions
3. Legislative Procedure
4. Bicameralism
5. Federation with a strong Center
Select the correct answer using the codes
below:
(a) 1, 2 and 4 only
(b) 1, 3, 4 and 5 only
(c) 1 and 2 only
(d) 1, 2, 3, 4 and 5 only
Consider the following statements:
1. Besides the Directive mentioned in
the
Part
IV
of
the
Indian
Constitution, there are certain other
directives addressed to the state, for
example Article 351, which are
justiciable in the Court of Law.
2. Doctrine
of
Harmonious
Construction is part of the Directive
Principles of State Policy mentioned
in the Indian Constitution.
Which of the following statements given
above is/are correct?
(a) 1 only
(b) 2 only
(c) Both 1 and 2
(d) Neither 1 nor 2
Which of the following statements is/are
correct with regard to proclamation of
financial emergency in India?
1. It can be extended to indefinite
period with an approval of the
Parliament for every six months.
2. A
resolution
approving
the
proclamation of financial emergency
is to be passed by either house of
Parliament by simple majority.

P a g e | 15
ias.analog@gmail.com

3.

89.

90.

91.

The President may issue directions


for reduction of salaries and
allowances of Supreme Court and
High Court Judges.
Select the correct answer using the codes
below:
(a) 1 only
(b) 2 and 3 only
(c) 1 and 3 only
(d) 1, 2 and 3
In order to be qualified for election to the
office of the President of India, a person
must fulfill certain conditions. Which of
the following are among those conditions?
1. He should be a citizen of India
2. He should have completed the age of
thirty-five years
3. He must have qualified for election
as a member of the Rajya Sabha.
4. He must not hold the office of profit
even under any local or another
which is subjected to government
control.
Select the correct answer using the codes
below:
(a) 1 and 2 only
(b) 2 and 3 only
(c) 1, 2 and 4 only
(d) 1, 3 and 4 only
In case of any dispute related with
treaties executed before the enforcement
of the Indian Constitution, under which
Jurisdiction of Supreme Court of India
litigant can file a case before it?
(a) Original jurisdiction
(b) Appellate Jurisdiction
(c) Advisory Jurisdiction
(d) Writ Jurisdiction
Consider
the
following
statements
regarding the Governor of an Indian state:
1. He can nominate a member to the
state legislative assembly from
Anglo-Indian community
2. He can make advances out of the
Contingency Fund of the state to
meet any unforeseen expenditure
3. He is chief spokesman of the state
government.
Select the correct answer using the codes
below:

92.

93.

ANALOG IAS INSTITUTE The Right Choice of Achievers.


www.analogeducation.in

(a) 1 and 2 only


(b) 2 and 3 only
(c) 1 and 3 only
(d) 1, 2 and 3
Which of the following statements
regarding the Union Public Service
Commission(UPSC) are correct?
1. The Chairman and members of the
commission shall hold office for a
term of six years or until they attain
the age of 65 years, whichever is
earlier.
2. The Chairman of UPSC (on ceasing
to hold office) is not eligible for
further employment in government
of India or a state.
3. The
entire
expenses
including
salaries, allowances and pensions of
the chairman and members of the
UPSC
is
charged
on
the
Consolidation Fund of India.
Select the correct answer using the codes
below:
(a) 1 and2 only
(b) 2 and 3 only
(c) 1 and 3 only
(d) 1, 2 and 3
Consider
the
following
statements
regarding
the
Central
Vigilance
Commission of India:
1. The Chairman and members of CVC
are appointed by President on the
recommendation of a three member
committee consisting of the Prime
Minister as its head, Central Home
Minister and leader of opposition in
Lok Sabha.
2. The Chairman and members hold
office for a term of six years or until
they attain the age 62 years
whichever is earlier.
3. It has all powers of civil court.
Select the correct answer using the codes
below:
(a) 1 and2 only
(b) 2 and 3 only
(c) 1 and 3 only
(d) 1, 2 and 3

P a g e | 16
ias.analog@gmail.com

94.

95.

96.

Which among the following statements


regarding quorum in a state legislative
assembly of India are correct?
1. It is the minimum number of
members in the assembly required to
be present in the house before it can
transact any business.
2. It is ten members or one-tenth of the
total number of members of the
house (including the presiding
officer) whichever is great.
3. If there is no quorum during meeting
of the house, it is duty of the
presiding officer either to adjourn or
to suspend the meeting until there is
quorum.
Select the correct answer using the codes
below:
(a) 1 and2 only
(b) 2 and 3 only
(c) 1 and 3 only
(d) 1, 2 and 3
Which among the following statements
regarding the removal process of a Judge
of a High court in India is/are correct?
1. A removal motion signed by 100
members (in case of Loksabha) or
50 members (in case of Rajya Sabha)
is to be given to the Speaker/
Chairman.
2. The Speaker/ Chairman has to
admit the motion, any one of them
cannot refuse it.
3. If the motion is admitted then the
speaker/ Chairman is to constitute a
10 member committee to investigate
charges.
Select the correct answer using the codes
below:
(a) 1 only
(b) 2 and 3 only
(c) 1 and 3 only
(d) 1, 2 and 3
Consider
the
following
statements
regarding
Committee
on
Public
Undertakings of the Parliament in India:
1. All the members are from Lok Sabha.
2. The Chairman of the committee is
appointed by the Leader of the house

97.

98.

99.

ANALOG IAS INSTITUTE The Right Choice of Achievers.


www.analogeducation.in

and Leader of opposition in Lok


Sabha.
3. Its recommendations are binding on
the ministers.
Which of the statements given above are
correct?
(a) 1 and 2 only
(b) 2 and 3 only
(c) 1 and 3 only
(d) None of the above
Consider
the
following
statements
regarding the Constitution rights of
Children in India:
1. The state shall provide free and
compulsory education to all children
of the age of six to fourteen years.
2. No child below the age of fourteen
years shall be employed in any
factory,
mine
or
hazardous
occupation.
3. Every child below ten years of age
whose family is living below the
poverty line is provided free food by
the state.
Select the correct answer using the codes
below:
(a) 1 and 2 only
(b) 2 and 3 only
(c) 1 and 3 only
(d) 1, 2 and 3
Which of the statements regarding
Central Administrative Tribunal(CAT) in
India are correct?
1. Its members are drawn from both
judicial and administrative streams.
2. It works under the administrative
control of Department of Personnel
and Training.
3. It is not guided by the principles of
natural justice.
Select the correct answer using the codes
below:
(a) 1 and 2 only
(b) 2 and 3 only
(c) 1 and 3 only
(d) 1, 2 and 3
Cabinet Secretariat in India performs
which among the following functions?
1. It provides secretarial assistance to
the Cabinet Committees.
P a g e | 17
ias.analog@gmail.com

2.

It keeps the President, Vice President


and all the Central Ministers
informed of the main activities of the
Central Government.
3. It functions as the chief coordination
agency in the Central Government.
Select the correct answer using the codes
below:
(a) 1 and 2 only
(b) 2 and 3 only
(c) 1 and 3 only
(d) 1, 2 and 3
100. Consider
the
following
statements
regarding Inner Line Permit (IPL) system:
1. It is an official travel document
issued by Indian Government to
allow inward travel of an Indian

citizen into a protected or restricted


area for a limited period.
2. It is issued under the Bengal Eastern
Frontier Regulation, 1873 and
currently it is in force in Arunachal
Pradesh, Nagaland, Manipur and
Meghalaya.
3. Recently Union Government has
rejected the demand to introduce it
in Mizoram.
Which of the states given above is/are
correct?
(a) 1 only
(b) 2 only
(c) 1 and 2 only
(d) 1, 2 and 3

*****

ANALOG IAS INSTITUTE The Right Choice of Achievers.


www.analogeducation.in

P a g e | 18
ias.analog@gmail.com

ANALOG IAS INSTITUTE The Right Choice of Achievers.


www.analogeducation.in

P a g e | 19
ias.analog@gmail.com

ANALOG IAS INSTITUTE The Right Choice of Achievers.


www.analogeducation.in

P a g e | 20
ias.analog@gmail.com

ANALOG IAS INSTITUTE


2nd Floor, 1-2-288/32, Indira Park 'X' Roads, Domalguda, Hyderabad 500029
Ph. No: 040-64590440, 9912441137
website: www.analogeducation.in

A2

PRELIMS TEST SERIES 2016

Indian Polity - KEY & EXPLANATION


Number of Questions: 100

1. Ans: D
Exp: The Commission includes 1 Chief
Information Commissioner (CIC) and not more
than 10 Information Commissioners (IC) who are
appointed by the President of India.
Candidates for CIC/IC must be persons of
eminence in public life with wide knowledge and
experience in law, science and technology, social
service, management, journalism, mass media or
administration and governance.
CIC/IC shall not be a Member of Parliament or
Member of the Legislature of any State or Union
Territory. He shall not hold any other office of
profit or connected with any political party or
carrying on any business or pursuing any
profession.
2. Ans: B
Exp: CIC shall be appointed for a term of 5 years
from date on which he enters upon his office or
till he attains the age of 65 years, whichever is
earlier. CIC is not eligible for reappointment.
Same goes for CAG, chairman UPSC and CEC.
Other members of Election Commission and
UPSC are re-eligible for the post of chairman or
chief.
3. Ans: D
Exp: Under the Cabinet Mission Plan of 1946,
elections were held for the first time for the
Constituent Assembly. The Constitution of India
was drafted by the Constituent Assembly, and it
was implemented under the Cabinet Mission Plan
on 16 May 1946. The members of the Constituent
Assembly were elected by the provincial
assemblies by a single, transferable-vote system
of proportional representation. The total
membership of the Constituent Assembly was
389: 292 were representatives of the states, 93
represented the princely states and four were
from the chief commissioner provinces of Delhi,
Ajmer-Mewar, Coorg and British Baluchistan.

4. Ans: C
Exp: The Central Vigilance Commissioner or any
Vigilance Commissioner can be removed from his
office only by order of the President on the ground
of proved misbehavior or incapacity after
the Supreme Court, on a reference made to it by
the President, has, on inquiry, reported that the
Central Vigilance Commissioner or any Vigilance
Commissioner, as the case may be, ought to be
removed. The President may suspend from office,
and if deem necessary prohibit also from
attending the office during inquiry, the Central
Vigilance Commissioner or any Vigilance
Commissioner in respect of whom a reference has
been made to the Supreme Court until the
President has passed orders on receipt of the
report of the Supreme Court on such reference.
The President may, by order, remove from office
the Central Vigilance Commissioner or any
Vigilance Commissioner if the Central Vigilance
Commissioner or such Vigilance Commissioner,
as the case may be:

Is adjudged an insolvent; or

Has been convicted of an offence which, in


the opinion of the Central Government,
involves moral turpitude; or

Engages during his term of office in any


paid employment outside the duties of his
office; or

Is, in the opinion of the President, unfit to


continue in office by reason of infirmity of
mind or body; or

Has acquired such financial or other


interest as is likely to affect prejudicially
his functions as a Central Vigilance
Commissioner
or
a
Vigilance
Commissioner
5. Ans: C
Exp: The following broad categories of criminal
cases are handled by the CBI:

1.

Cases of corruption and fraud


committed by public servants of all
Central Govt. Departments, Central
Public Sector Undertakings and
Central Financial Institutions.

2.

Economic crimes, including bank


frauds, financial frauds, Import
Export
&
Foreign
Exchange
violations, large-scale smuggling of
narcotics, antiques, cultural property
and smuggling of other contraband
items etc.

3.

Special Crimes, such as cases of


terrorism, bomb blasts, sensational
homicides, kidnapping for ransom
and crimes committed by the
mafia/the underworld.

6. Ans: D
Exp: 1919 act brought dyarchy to provinces,
there
by
relaxing
central
control
over
provinces...despite dyarchy majority of decisions
and final say was in e hands of viceroy.
7. Ans: A
Exp: As per the amendment the changes done to
Constitution are:In Part III of the Constitution, after words or
unions the words Cooperative Societies was
added.
In Part IV a new Article 43B was inserted, which
says: The state shall endeavour to promote
voluntary formation, autonomous functioning,
democratic control and professional management
of the co-operative societies.
After Part IXA of the constitution, a Part IXB was
inserted to accommodate state vs centre roles.
Salient features Part IXB
It makes Right to form cooperatives is
a fundamental right.
Reservation of one seat for SC/ST and two seats
for women on the board of every co-operative
society. Cooperatives could set up agency which
would oversee election.
Uniformity in the tenure of Cooperative Board of
Directors.
Provisions for incorporation, regulation and
winding up of co-operative societies based on the
principles of democratic process and specifying
the maximum number of directors as twenty-one.
Providing for a fixed term of five years from the
date of election in respect of the elected members
of the board and its office bearers;

Providing for a maximum time limit of six months


during which a board of directors of co-operative
society could be kept under suspension;
Providing for independent professional audit;
Providing for right of information to the members
of the co-operative societies;
Empowering the State Governments to obtain
periodic reports of activities and accounts of cooperative societies; which have individuals as
members from such categories;
Providing for offences relating to co-operative
societies and penaltiesin respect of such offences.
8. Ans: C
Exp: The enactment of Administrative Tribunals
Act in 1985 opened a new chapter in the sphere
of administering justice to the aggrieved
government servants. Administrative Tribunals
Act owes its origin to Article 323-A of the
Constitution
which
empowers
Central
Government to set-up by an Act of Parliament,
Administrative Tribunals for adjudication of
disputes and complaints with respect to
recruitment and conditions of service of persons
appointed to the public service and posts in
connection with the affairs of the Union and the
States. In pursuance of the provisions contained
in the Administrative Tribunals Act, 1985, the
Administrative Tribunals set-up under it exercise
original jurisdiction in respect of service matters
of employees covered by it. As a result of the
judgement dated 18 March 1997 of the Supreme
Court, the appeals against the orders of an
Administrative Tribunal shall lie before the
Division Bench of the concerned High Court.
9. Ans: C
Exp: All India services controlled by the central
and state governments. The ultimate control lies
with the central government while the immediate
control vested in the state government. Any
disciplinary action against these officers can only
be taken by the central government.
The personnel of Central services (even though
are posted in states) work under the exclusive
jurisdiction of the central government.
10. Ans: A
Exp: The constitution does not specify the castes
or tribes which are to be called the SCs or the
STs. It leaves to the president the power to specify
as to what castes or tribes in each state and UT
are to be treated as so in case of states
concerned, the president issue the notification

ANALOG IAS INSTITUTE The Right Choice of Achievers.


www.analogeducation.in

Page |2
ias.analog@gmail.com

after consulting the governor of the state. But,


any inclusion or exclusion of any cast or tribe
from presidential notification can be done only by
the Parliament and not by subsequent
presidential notification.
11. Ans: D
Exp: The Election commissioner registers political
parties for the purpose of elections and grants
them recognition as national or state parties on
the basis of their poll performance.
A political party is called a national party if :- (i) it
secures at least six percent(6%) of the valid votes
polled in any four or more states, at a general
election to the House of the People or, to the State
Legislative Assembly; and (ii) in addition, it wins
at least four seats in the House of the People from
any State or States. OR it wins at least two
percent (2%) seats in the House of the People i.e.
Lok Sabha, and these members are elected from
at least three different States.
Election commission (EC) has scratched all the
previous criteria for being a state party.
Previously, for being a state party a political party
had to secure 6% of the total valid votes polled &
in addition had to win some seats or atleast three
percent of the total number of seats or three seats
in the Legislative Assembly, whichever is more.
Alternatively, a political party should secure at
least six percent of the total valid votes polled in a
State during a general election to Lok Sabha and
win at least one seat in the Lok Sabha from that
State, or the party should win at least one seat in
the Lok Sabha for every 25 seats or any fraction
thereof allotted to that State.
Now, according to EC, any political party in the
state which is able to secure 8% of the total Valid
votes polled (by all of its candidates together) will
be hence forth recognized as the state party,
irrespective of the number of seats it secures.
Also, if the party wins 6% of total Votes polled
(not only Valid votes) it would qualify for that. In
addition party should win one Assembly seat per
25 seats of that state.
12. Ans: A
Exp: The grounds for disqualification under the
Anti-Defection Law's are as follows.
(a) If an elected member voluntarily gives up his
membership of a political party;
(b) If he votes or abstains from voting in such
House contrary to any direction issued by

his political party or anyone authorised to


do so, without obtaining prior permission.
As a pre-condition for his disqualification, his
abstention from voting should not be condoned
by his party or the authorised person within 15
days of such incident.
Finally the 91st Constitutional Amendment Act,
2003, changed this. So now at least two-thirds of
the members of a party have to be in favour of a
"merger" for it to have validity in the eyes of the
law. "The merger of the original political party or a
member of a House shall be deemed to have
taken place if, and only if, not less than twothirds of the members of the legislature party
concerned have agreed to such merger," states
the Tenth Schedule.
13. Ans: B
Exp:
1

President Pranab Mukherjee

Vice-President Mohammad Hamid Ansari

Prime Minister Narendra Modi

Governors of states of India (within their


respective States)

Former Presidents PratibhaPatil

5A

Deputy Prime Minister vacant

Speaker of Lok Sabha Sumitra Mahajan,


Chief Justice of India
Cabinet Ministers of the Union
Chief Ministers of States (within their
respective States)
Deputy Chairman of NITI Aayog

Former Prime Ministers Atal Bihari


Vajpayee, H. D. Deve Gowda, Manmohan
Singh
Leaders of the Opposition in the Rajya
Sabha and Lok Sabha Ghulam Nabi
Azad and vacant respectively

7A

Holders of the Bharat Ratna Amartya


Sen, LataMangeshkar, C.
N.
R.
Rao, Sachin
Tendulkar, Atal
Bihari
Vajpayee

Ambassadors
Extraordinary
and
Plenipotentiary and High Commissioners

ANALOG IAS INSTITUTE The Right Choice of Achievers.


www.analogeducation.in

Page |3
ias.analog@gmail.com

of Commonwealth countries accredited to


India
Chief Ministers of States (when outside
their respective States)
Governors of States (when outside their
respective States)
9

Judges
of Supreme
India (Justices of India)

Court

Chairman, Union
Public
Commission Deepak Gupta
9A

of
Service

Chief Election Commissioner Syed Nasim


Ahmad Zaidi
Comptroller and Auditor General Shashi
Kant Sharma
Deputy Chairman, Rajya Sabha P. J.
Kurien
Deputy Chief Ministers of States

10

Deputy Speaker
Thambidurai

of

Lok

Sabha

M.

Members of the Planning Commission


Ministers of States of the Union

11

Lieutenant
Governors
within
respective Union Territories

their

Attorney General of India MukulRohatgi


Cabinet Secretary Pradeep Kumar Sinha

14. Ans: C
Exp: Article 324- there is only one general
electoral roll for every territorial constituency for
elections to the Parliament and the state
legislature. Thus, the constitution has abolished
the system of communal representation and
separate electorates which led to the partition of
the country.
The state legislatures, apart from Parliament can
also make provision with respect to all matters
relating to the elections to the state legislature
including the preparation of electoral rolls and all
other matters necessary for securing their due
constitution.
15. Ans: D
Exp: FR clearly establish a secular, equitable,
just and non-exploitative social fabric in India.
FD provide a secular, fraternity and a patriotic
aspect to the Indian citizens by making them

respect the constitution, national flag, asking


them render services to the nation and mankind
etc.
DPSP does so by providing for a more equitable
work culture in men and women, minimizing
inequalities in social and economic status:
establishing the ideal of a welfare society.
16. Ans: D
Exp: Citizenship clauses need a simple majority
only in the Parliament.
The same is true for changing the boundary of
states and for changing number of judges.
17. Ans: C
Exp: Features
The features
of
Parliamentary
form
of
Government has been discussed below:
1. Existence of a Titular or Constitutional
Ruler: The first characteristic feature of the
parliamentary system is the existence of a
Titular of Constitutional Ruler. Legally the
administration of all the affairs of the state
is conducted by the head of the state. In
reality, however, the administration is
carried by the Council of Ministers. The
Monarch or the President, as the case may
be, is the head of the state, but not the head
of the government.
2. Absence of Separation of Powers: In the
parliamentary system the principle of
separation of powers is not adopted. Here
the three departments of government work
in close, intimate contact, sharing some of
the powers and functions of one another.
3. Main Role of the Lower House in
Ministry-formation: In the parliamentary
government the lower house of the
legislature, i.e., the popular chamber plays a
vital role in the formation of the ministry.
The leader of the party or alliance which
wins the majority in this house is appointed
the Prime Minister or Chancellor. The
constitutional ruler appoints the other
members of the ministry on his advice.
4. Responsibility to the Legislature: In such
a system the Cabinet or Ministry has to
remain responsible to the legislature for all
its activities and policies. In countries
having bi-cameral legislatures, the Cabinet
remains responsible to the lower house
composed of the peoples representatives.

ANALOG IAS INSTITUTE The Right Choice of Achievers.


www.analogeducation.in

Page |4
ias.analog@gmail.com

5.

Collective Responsibility: The ministerial


responsibility to the legislature may again be
of two kinds:

Individual responsibility, and

Collective responsibility.
Individual responsibility means that the minister
in charge of a department must be answerable for
the activities of his department. But when the
ministers remain jointly or collectively responsible
to the legislature for the policies and activities of
the government, it is called collective
responsibility. Since no individual minister can
unilaterally perform any business of government
without the consent of the Cabinet, the entire
Ministry or Cabinet has to remain accountable for
the errors of the minister concerned.
6. Intimate
relationship
between
the
Legislature and the Executive: In the
parliamentary
system
an
intimate
relationship exists between the executive
and the legislative departments. So they
can easily control each other. The leaders
of the majority party or alliance in the
legislature become the members of the
Cabinet or Ministry. Naturally, the
ministers can easily extend their influence
on the legislature. Consequently, the
programs and policies of the Cabinet are
backed by a majority inside the legislature.
7. Leadership of the Prime Minister: The
leadership of the Prime Minister is another
major feature of the parliamentary system.
The leader of the majority party in the
legislature becomes the Prime Minister.
Though, in theory, he is primus inter
pares, i.e. first among equals, in reality,
he possesses much greater power and
status than the other ministers. As the
undisputed leader of the majority party or
alliance in the legislature he plays the
most vital role in the determination and
execution of government policies. Indeed,
the success of parliamentary democracy
depends, to a great extent, on the
personality, efficiency and charisma of the
Prime Minister.
8. Existence of a Strong Opposition: The
existence of one or more strong and wellorganized opposition party or parties is the
hall-mark of the parliamentary system. By
criticizing the errors of the government,

the opposition can compel it to adopt


welfare measures and prevent it from
becoming despotic. Judged from this
angle, the opposition can be called the lifeforce of parliamentary democracy.
9. Cabinet Dictatorship: In the parliamentary
system of government the cabinet has to
perform manifold functions.
18. Ans: B
Exp: Article 74
(1)

There shall be a Council of Ministers with


the PM at the head to aid and advise the
President who shall, in the exercise of his
functions, act in accordance with such
advises. (The matter in bold letters is
added by the Forty-second Amendment of
the Constitution of India in the year 1977)

Provided that the President may require the


Council of Ministers to reconsider such advice,
either generally or otherwise, and the President
shall act in accordance with the advises tendered
after such reconsideration. (This para is added by
the Forty-fourth Amendment of the Constitution
of India in the year 1978)
(2)

The question if any, and if so what, advice


was tendered by Ministers to the President
shall not be inquired into in any court.
Duties of Prime Minister as respects the
furnishing of information to the President, etc It
shall be the duty of the Prime Minister
(a) To communicate to the President all
decisions of the council of Ministers
relating to the administration of the affairs
of the union and proposals for legislation;
(b) To furnish such information relating to the
administration of the affairs of the Union
and proposals for legislation as the
President may call for; and
(c) If the President so requires, to submit for the
consideration of the Council of Ministers
any matter on which a decision has been
taken by a Minister but which has not
been considered by the Council
19. Ans: D
Exp: Article 74
(1)

There shall be a Council of Ministers with


the PM at the head to aid and advise the
President who shall, in the exercise of his
functions, act in accordance with such
advises. (The matter in bold letters is

ANALOG IAS INSTITUTE The Right Choice of Achievers.


www.analogeducation.in

Page |5
ias.analog@gmail.com

added by the Forty-second Amendment of


the Constitution of India in the year 1977)
Provided that the President may require the
Council of Ministers to reconsider such advice,
either generally or otherwise, and the President
shall act in accordance with the advises tendered
after such reconsideration. (This para is added by
the Forty-fourth Amendment of the Constitution
of India in the year 1978)
(2)

The question if any, and if so what, advice


was tendered by Ministers to the President
shall not be inquired into in any court.
20. Ans: D
Exp:143. Power of President to consult Supreme
Court (1) If at any time it appears to the President
that a question of law or fact has arisen, or is
likely to arise, which is of such a nature and of
such public importance that it is expedient to
obtain the opinion of the Supreme Court upon it,
he may refer the question to that Court for
consideration and the Court may, after such
hearing as it thinks fit, report to the President its
opinion thereon
(2) The President may, notwithstanding anything
in the proviso to Article 131, refer a dispute of the
kind mentioned in the said proviso to the
Supreme Court for opinion and the Supreme
Court shall, after such hearing as it thinks fit,
report to the President its opinion thereon
21. Ans: D
Exp: Article 13 gives the judiciary the powers to
review the legislative and administrative rules
Article 19: Protection of certain rights regarding
freedom of speech, etc.
Article 21: Protection of life and personal liberty.
Article 32-The right to move the Supreme Court
by appropriate proceedings for the enforcement of
the rights conferred by this Part is guaranteed
22. Ans: A
Exp: Proportional representation is a type of
electoral system that decides the make-up of a
parliament by allocating seats on the basis of the
number of votes each party received. Although
there are many different types of PR, this is the
base requirement for a system to be described as
proportional.
Rather than the winner-take all approach of other
systems, PR ensures that votes carry equal
weight. To do this, multi-member constituencies
are used. This means that a single area elects
more than one representative. The size of this

area can vary according to the system, ranging


from the size of the whole country to a county or
local vicinity.
23. Ans: D
Exp: Article 370 of the Indian constitution is an
article that grants special autonomous status to
the state of Jammu and Kashmir. The article is
drafted in Part XXI of the Constitution, which
relates to Temporary, Transitional and Special
Provisions.
370. Temporary provisions with respect to the
State of Jammu and Kashmir
(1) Notwithstanding anything contained in this
Constitution,
(a) The provisions of article 238 shall not apply
now in relation to the state of Jammu and
Kashmir.
(b) The power of Parliament to make laws for the
said state shall be limited to
(i) Those matters in the Union List and
the Concurrent List which, in consultation with
the Government of the State, are declared by
the President to correspond to matters specified
in the Instrument of Accession governing the
accession of the State to the Dominion of India as
the matters with respect to which the Dominion
Legislature may make laws for that State; and
(ii) such other matters in the said Lists as, with
the concurrence of the Government of the State,
the President may by order specify.
Explanation: For the purpose of this article, the
Government of the State means the person for the
time being recognized by the President on the
recommendation of the Legislative Assembly of
the State as the Sadr-i-Riyasat (now Governor) of
Jammu and Kashmir, acting on the advice of the
Council of Ministers of the State for the time
being in office.
(c) The provisions of article 1 and of this article
shall apply in relation to that State;
(d) Such of the other provisions of this
Constitution shall apply in relation to that State
subject to such exceptions and modifications as
the President may by order specify:
Provided that no such order which relates to the
matters specified in the Instrument of Accession
of the State referred to in paragraph (i) of subclause (b) shall be issued except in consultation
with the Government of the State:

ANALOG IAS INSTITUTE The Right Choice of Achievers.


www.analogeducation.in

Page |6
ias.analog@gmail.com

Provided further that no such order which relates


to matters other than those referred to in the last
preceding proviso shall be issued except with the
concurrence of that Government.
(2) If the concurrence of the Government of the
State referred to in paragraph (ii) of sub-clause (b)
of clause (1) or in the second provison to subclause (d) of that clause be given before the
Constituent Assembly for the purpose of framing
the Constitution of the State is convened, it shall
be placed before such Assembly for such decision
as it may take thereon.
(3) Notwithstanding anything in the foregoing
provisions of this article, the President may, by
public notification, declare that this article shall
cease to be operative or shall be operative only
with such exceptions and modifications and from
such
date
as
he
may
specify:
Provided that the recommendation of the
Constituent Assembly of the State referred to in
clause (2) shall be necessary before the
President issues such a notification
24. Ans: B
Exp: Article 74(1) clearly prescribes that the
president is bound by the advice of the COM.
Whereas in the case of Governor, the respective
provision does not make the advice of the COM in
state binding on him.
25. Ans: D
Exp: Schedule 5 and 6 areas not only defined and
demarcated by the constitution, but also several
administrative provisions like tribal advisory
council; powers of gram sabhas in those areas.
center-state relations have been given in so much
detail containing even the minutest of provisions
of taxation, administration and finances.
26. Ans: D
Exp: Laws made under Articles 2 and 3 to
provide for the amendment of the First and the
Fourth Schedules and supplemental, incidental
and consequential matters
(1) Any law referred to in Article 2 or Article 3
shall contain such provisions for the amendment
of the First Schedule and the Fourth Schedule as
may be necessary to give effect to the provisions
of the law and may also contain such
supplemental, incidental and consequential
provisions
(including
provisions
as
to
representation in Parliament and in the
Legislature or Legislatures of the State or States

affected by such law) as Parliament may deem


necessary
(2) No such law as aforesaid shall be deemed to
be an amendment of this Constitution for the
purposes of Article 368.
The supreme court in 1969 ruled that Settlement
of boundary Dispute between India and other
country does not require a constitutional
amendment. It also done by executive action as it
does not involve cession of Indian territory to a
foreign country.
27. Ans: A
Exp: Article 5-8 conferred citizenship on each
person
who
met
the criteria
at
the
commencement of the Constitution. However it
contains neither any permanent nor any
elaborate provisions in this regard. It only
identifies the persons who became citizens of
India at its commencement. It does not deal with
the problems of acquisition or loss of citizenship
subsequent to its commencement. It leaves the
same to Parliament.
28. Ans: A
Exp: Under the Cabinet Mission Plan of 1946,
elections were held for the first time for the
Constituent Assembly. The Constitution of India
was drafted by the Constituent Assembly, and it
was implemented under the Cabinet Mission Plan
on 16 May 1946. The members of the Constituent
Assembly were elected by the provincial
assemblies by a single, transferable-vote system
of proportional representation. The total
membership of the Constituent Assembly was
389: 292 were representatives of the states, 93
represented the princely states and four were
from the chief commissioner provinces of Delhi,
Ajmer-Mewar, Coorg and British Baluchistan.
29. Ans: D
Exp: refer to the provisions of the Government of
India Act 1935, to understand how these
provisions affected Indian Constitution.
30. Ans: B
Exp: Features of Parliamentary Government
1. Nominal or Titular Head:
In a Parliamentary form of government, there two
heads, namely, nominal and real. The nominal
head is one who, though head of the state, is not
head of government. His powers are more
apparent than real. He may be hereditary or
elected.

ANALOG IAS INSTITUTE The Right Choice of Achievers.


www.analogeducation.in

Page |7
ias.analog@gmail.com

The British Queen is not elected. She got the


throne on the heredity. But the President of India
who is also a nominal head has been el
Parliamentary-government the real powers are
exercised by a Council of Ministers by a Prime
Minister.
2. Collective Responsibility and Individual
Responsibility:
The C Ministers is collectively responsible to the
lower house of the legislature. It policy decisions
collectively and it collectively goes out of office
when it loses thee of the lower house of the
legislature. A minister may express his
disagreement policy when it is discussed in the
cabinet meeting, but he has to defend and
support the cabinet takes the decision. A minister
is also individually responsible to the P for the
acts of omission and commission of his
department.
3. Political Homogeneity:
The ministers, normally being members of
political party, share the same ideology and
approach. Even when there is a c government,
the ministers are committed to a common
minimum programme. B single party government
and a coalition government, there is a fair
amount of h and cooperation among the
ministers. However, a single party government is
homogeneous than a multy-party coalition
government,
4.
Harmony
between
Executive
and
Legislature:
In a Parliamentary government the ministers are
drawn from the legislature. As ministers, they are
part of the executive. They also remain members
of the legislature. Thus the dual identity of
ministers con to a harmonious relationship
between the executive and the legislature.
5. Rigidity of Party Discipline:
In a Parliamentary government, the party
discipline is rigid. The members of a political
party whether in power or in opposition are
required to defend and support the stand of their
party on any issue both in the legislature and
outside.
6. Leadership of the Prime Minister:
The Prime Minister is the leader of the C of
Ministers. On his advice, the ministers are
appointed and dropped. They stay in during his

pleasure. He presides over the meetings of the


cabinet. He exercises preponderant influence in
domestic policy as well as foreign policy.
He is more powerful and important than any
other member of the cabinet. It has been rightly
observed that "he (Prime Mi is central to its
(ministry's) birth, central to its life and central to
its death."
31. Ans: A
Exp: India Independence Act of 1947: Legislation
for the new dominions:
1.

The existing legislative setup was allowed


to continue as Constitution making body
as well as a legislature. (Temporary
Provisions as to the Government of Each
New Dominion.)

2.

The legislature of each dominion was given


full powers to make laws for that
dominion,
including
laws
having
extraterritorial operation.

3.

No Act of Parliament of UK passed after


the appointed date would be extended to
the territories of new dominions.

4.

No law and provision of any law made by


the legislature of the new dominions shall
be void or inoperative on the ground that it
is repugnant to the law of England.

5.

The Governor-General of each dominion


had full powers to give assent in His
Majestys name to any law of the
legislature. [Configuration of Pakistans
Constitution Assembly (CAP I): 69
members of the central legislature + 10
immigrant members= 79]
32. Ans: D
Exp: The Drafting Committee for framing the
constitution was appointed on 29 August 1947.
The committee comprised of a chairman and six
other members. In addition a constitutional
advisor was also appointed.
The committee members were:
Dr B. R. Ambedkar - Chairman
K M Munshi (Ex- Home Minister, Bombay)
Alladi Krishna swamyIyer (Ex- Advocate General,
Madras State)

ANALOG IAS INSTITUTE The Right Choice of Achievers.


www.analogeducation.in

Page |8
ias.analog@gmail.com

N Gopalaswamin Ayengar (Ex-Prime Minister,


J&K, member of Nehru Cabinet)
B L Mitter (Ex-Advocate General, India)
Md. Saadullah (Ex- Chief Minister of Assam,
Muslim League member)
D P Khaitan (lawyer).
33. Ans: D
Exp: 5 and 6 schedules
Article 370
And article 371.
34. Ans: A
Exp: Presidents Rule
Grounds of Imposition
Article 355 imposes a duty on the Centre to
ensure that the government of every state is
carried on in accordance with the provisions of
the Constitution. It is this duty in the
performance of which the Centre takes over the
government of a state under Article 356 in case of
failure of constitutional machinery in state. This
is popularly known as Presidents Rule. It is also
known as State Emergency or Constitutional
Emergency.
The Presidents Rule can be proclaimed under
Article 356 on two groundsone mentioned in
Article 356 itself and another in Article 365:
1.

Article 356 empowers the President to issue


a proclamation, if he is satisfied that a
situation has arisen in which the
government of a state cannot be carried on
in accordance with the provisions of the
Constitution. Notably, the president can
act either on a report of the governor of the
state or otherwise too (i.e., even without
the governors report).

2.

Article 365 says that whenever a state fails


to comply with or to give effect to any
direction from the Centre, it will be lawful
for the president to hold that a situation
has arisen in which the government of the
state cannot be carried on in accordance
with the provisions of the Constitution.
35. Ans: D
Exp: In any Written Constitution, Preamble is needed
for following four reasons or may also be said to be
the main four Ingredients or Components of a Written
Constitution -

1.

It declares the ultimate source of authority


such as "We the people of India"
2. It declares the type of Government such as
"Sovereign, Socialist, Secular, Democratic
Republic
3. It decides the objective of administration
such as social, economic and political
justice.
4. It declares the date of adoption of
Constitution that is 26th November, 1949.
36. Ans: D
Exp: With reference to the government in India,
the two terms namely Union of India and
Territory of India have specific meanings. While
the former stands for the states that share federal
powers with the Union Governments, the latter
include not only the states but all other units like
the Union Territories also or, in other words, the
territory of India includes a larger area as
compared to the union of India. i.e. the territory of
India covers the entire territory over which Indian
sovereignty is exercised while Union of India
covers only the federal system.
Being a sovereign state, India can acquire foreign
territories according to the modes recognized by
international law i.e cession, occupation,
conquest or subjugation.
Article 2: Grants two powers to the Parliament
namely:
1)

The power to admit into the Union of


India new states; and

2)

The power to establish new states

Broadly, the first refers to the admission of


states which are already in existence while the
second refers to the establishment of states
which are not in existence before. The latest
addition was that of the state of Sikkim by the
35th and 36th Constitutional Amendments in
1974 and 1975. This was done at the request
of the Sikkim Assembly and after the approval
the people of Sikkim through a referendum.
37. Ans: D
Exp: Executive and legislature of Union and
states include union & state governments along
with Parliament and state legislatures. The
President of India and Governors of states can
also be referred as State as they are a part of the
executive. The term government also includes

ANALOG IAS INSTITUTE The Right Choice of Achievers.


www.analogeducation.in

Page |9
ias.analog@gmail.com

any department of government or any institution


under its control. The Income Tax Department
and the International Institute for Population
Sciences could be cited as examples.
Local authorities, as used in the definition, refer
to municipalities, panchayats or similar
authorities that have the power to make laws &
regulations and also enforce them. The
expression Other authorities could refer to any
entity that exercises governmental or sovereign
functions.
Both statutory and non-statutory bodies can be
considered as a State provided they get financial
resources from the government and have deep
pervasive control of government and with
functional characters. ONGC, Delhi Transport
Corporation, IDBI, and Electricity Boards are
referred as a State. However, entities such as
NCERT cannot be considered a State as they are
not substantially financed by the government and
the government control is not pervasive
38. Ans: A
Exp: Freedom of speech and expression is the
most basic of all freedoms granted to the citizens
of India. J Patanjali Shastri has said in the case
of Romesh Thaper vs State of Madras AIR 1950
SC that freedom of speech and that of the press
lay at the foundation of a democratic society, for
without free political discussions, no public
education is possible, which is so important for
the
proper
functioning
of
the
govt.
It allows us to freely express our ideas and
thoughts through any medium such as print,
visual, and voice. One can use any
communication medium of visual representation
such as signs, pictures, or movies. Freedom of
speech would amount to nothing if it were not
possible to propagate the ideas. Thus, the
freedom of publication is also covered under
freedom of speech. Freedom of speech serves 4
purposes

allows an individual to attain self


fulfillment.

assists in the discovery of truth.

it strengthens the capacity of a person to


make decisions.

it facilitates a balance between stability


and social change.
This right is not only about communicating your
ideas to others but also about being able to

publish and propagate other people's views as


well. Thus, freedom of speech and expression is
linked to the people's right to know. Freedom of
speech and expression is a broad term and
encompasses several things. The following are
important cases that have determined the extent
of this right from time to time.
Freedom of press
Freedom of commercial advertisements.
Right against tapping of telephonic conversation
Right to telecast
Right to know about government activities.
Right against bandh called by a political party or
organization
Freedom of silience.
Right to demonstration or picketing but not
strike.
39. Ans: A
Exp: Traffic in human beings and begar and
other similar forms of forced labour are prohibited
and any contravention of this provision shall be
an offence punishable in accordance with law. (2)
Nothing in this article shall prevent the State from
imposing compulsory service for public purposes,
and in imposing such service the State shall not
make any discrimination on grounds only of
religion, race, caste or class or any of them.
Slavery in its ancient form may not so much be a
problem in every State today but its newer forms
which are labelled in the Indian Constitution
under the general term exploitation are no less a
serious challenge to human freedom and
civilisation. It is in this view that our
Constitution, instead of using the word slavery
uses the more comprehensive expression traffic
in human beings which includes a prohibition
not only of slavery but also of traffic in women or
children or the crippled, for immoral or other
purposes.
In this regard, the Bonded Labour System
(abolition) act 1976, Equal Remuneration Act,
1976, Contract labour Act, 1970 and Minimum
wages Act, 1948 were made.
40. Ans: D
Exp: Freedom of conscience and free profession,
practice and propagation of religion
(1)
Subject to public order, morality and health
and to the other provisions of this Part, all
persons are equally entitled to freedom of
conscience and the right freely to profess,
practise and propagate religion

ANALOG IAS INSTITUTE The Right Choice of Achievers.


www.analogeducation.in

P a g e | 10
ias.analog@gmail.com

(2)

Nothing in this article shall affect the


operation of any existing law or prevent
the State from making any law
(a)
Regulating or restricting any economic,
financial, political or other secular activity
which may be associated with religious
practice;
(b)
Providing for social welfare and reform or the
throwing open of Hindu religious
institutions of a public character to all
classes
and
sections
of
Hindus
Explanation I The wearing and carrying of
kirpans shall be deemed to be included in
the profession of the Sikh religion
Explanation II In sub clause (b) of clause
reference to Hindus shall be construed as
including a reference to persons professing
the Sikh, Jaina or Buddhist religion, and
the
reference
to
Hindu
religious
institutions shall be construed accordingly
41. Ans: C
Exp: the difference between are:
1.

The Writ Jurisdiction of Supreme Court is


mentioned under Article 32 of the Indian
Constitution, while the Writ Jurisdiction of
High Courts is mentioned under Article
226 of the Indian Constitution.

2.

The High Courts have wider powers as


compare to Supreme Court in issuing writs.

3.

The Supreme Court can issue writ only in


case of violation of any of the fundamental
rights contained in Part-III of the
constitution, while the High Courts can
issue writs not only in case of violation of
fundamental rights but also in case of
violation of any legal rights of the citizens
provided that a writ is a proper remedy in
such cases, according to well-established
principles.

4.

Article 32 of the Constitution of India


imposes on the Supreme Court a duty to
issue the writs, whereas no such duty is
imposed on the High Courts by Art-226.

5.

The jurisdiction of the Supreme Court


extends all over the country, whereas that
of the High Courts only to the territorial
confines of the particular state and the
Union Territory to which its jurisdiction extends.

42. Ans: D
Exp: Article 27: No person shall be compelled to
pay any taxes, the proceeds of which are
specifically appropriated in payment of expenses
for the promotion or maintenance of any
particular religion or religious denomination.
This means that the taxes can be used for the
promotion or maintenance of all religions.
The provision prohibits only levy of taxes and not
a fee. This is because the purpose of a fee is to
control secular administration of religious
institutions and not to promote or maintain
religion.
43. Ans: C
Exp: It means that a law that is duly enacted by
legislature or the concerned body is valid if it has
followed the correct procedure. Following this
doctrine means that, a person can be deprived of
his life or personal liberty according to
the procedure
established
by
law. So,
if
Parliament pass a law, then the life or personal
liberty of a person can be taken off according to
the provisions and procedures of the that law.
This doctrine has a major flaw. It does not seek
whether the laws made by Parliament is fair, just
and not arbitrary. Procedure established by law
means a law duly enacted is valid even if its
contrary
to
principles
of
justice
and
equity. Strictly following procedure established by
law may raise the risk of compromise to life and
personal liberty of individuals due to unjust laws
made by the law making authorities. It is to avoid
this situation, SC stressed the importance of due
process of law.
44. Ans: A
Exp: these are the constitutional instructions to
the
state
in
legislative,
executive
and
administrative matters.
In the Part IV, unless the context otherwise
requires, the State has the same meaning as in
Part III.
45. Ans: C
Exp: go through the act.
46. Ans: D
Exp: Salient features Part IXB
It makes Right to form cooperatives is
a fundamental right.
Reservation of one seat for SC/ST and two seats
for women on the board of every co-operative
society. Cooperatives could set up agency which
would oversee election.

ANALOG IAS INSTITUTE The Right Choice of Achievers.


www.analogeducation.in

P a g e | 11
ias.analog@gmail.com

Uniformity in the tenure of Cooperative Board of


Directors.
Provisions for incorporation, regulation and
winding up of co-operative societies based on the
principles of democratic process and specifying
the maximum number of directors as twenty-one.
Providing for a fixed term of five years from the
date of election in respect of the elected members
of the board and its office bearers;
Providing for a maximum time limit of six months
during which a board of directors of co-operative
society could be kept under suspension;
Providing for independent professional audit;
Providing for right of information to the members
of the co-operative societies;
Empowering the State Governments to obtain
periodic reports of activities and accounts of cooperative societies; which have individuals as
members from such categories;
Providing for offences relating to co-operative
societies and penalties in respect of such
offences.
47. Ans: B
Exp: As per Swaran Singh committee including
duty to pay taxes should have been a
fundamental duty as well as fundamental right,
but it was not agreed upon.
As per the Direct taxation laws; income and other
kind of taxes are a legal obligation for an Indian
citizen as he uses public service, receives social
and political security etc.
48. Ans: B
Exp: It shall be the duty of every citizen of India:
1.

To abide by the Constitution and respect


its ideas and institutions, the National
Flag and the National Anthem;

2.

To cherish and follow the noble ideals


which inspired our national struggle for
freedom;

3.

To uphold and protect the sovereignty,


unity and integrity of India;

4.

To defend the country and render national


service when called upon to do so;

5.

To promote harmony and spirit of common


brotherhood among all the people of India,
transcending religious, linguistic, regional
or sectional diversities, to renounce
practices derogatory to the dignity of
women;

6.

To value and preserve the rich heritage of


our composite culture;

7.

To protect and improve the natural


environment including forests, lakes, river,
and wildlife and to have compassion for
living creatures;

8.

To
develop
the
scientific
temper,
humanism and spirit of inquiry and
reform;

9.

To safeguard public property and to abjure


violence;

10.

To strive towards excellence in all spheres


of individual and collective activities so
that the nation constantly rises to higher
levels of endeavor and achievement;

11.

To provide opportunities for education to


his child or, as the case may be, ward
between age of 6 and 14 years;
49. Ans: B
Exp: An amendment of the Constitution can be
initiated only by the introduction of a Bill in
either House of Parliament. The Bill must then be
passed in each House by a majority of the total
membership of that House and by a majority of
not less than two-thirds of the members of that
House present and voting. There is no provision
for a joint sitting in case of disagreement between
the two Houses. The Bill, passed by the required
majority, is then presented to the President who
shall give his assent to the Bill. If the amendment
seeks to make any change in any of the
provisions mentioned in the proviso to article 368
50. Ans: A
Exp: There are certain provisions which require
simple majority for amendments. They can be
amended by the ordinary law making process.
They include
(a) Formation of new states and alteration of
areas, boundaries or names of existing
ones
(b) Creation or abolition of Legislative Councils
in the states
(c) Administration and control of scheduled
areas and scheduled Tribes
(d) The salaries and allowances of the Supreme
Court and High Court Judges
(e) Laws regarding citizenship etc.
It is significant that the laws passed by
Parliament to change the above provisions would

ANALOG IAS INSTITUTE The Right Choice of Achievers.


www.analogeducation.in

P a g e | 12
ias.analog@gmail.com

not be deemed to be amendments of the


Constitution for the purpose of Article 368.
51. Ans: D
Exp: Cabinet collective responsibility is related to
the fact that, if a vote of no confidence is passed
in Lok Sabha (not Parliament), the government is
responsible collectively, and thus the entire
government resigns. The consequence will be that
a new government will be formed, or parliament
will dissolve and a general election will be called.
Cabinet collective responsibility is not the same
as individual ministerial responsibility, which
states
that ministers are
responsible
and
therefore culpable for the running of their
departments.
52. Ans: A
Exp: There are two ways in which the Lok Sabha
can be dissolved. First, when the leader of the
majority (the ruling party) dissolves the Lok
Sabha. While in this situation, the prerogative
and timing is that of the leader of the majority, he
has to dissolve the Lok Sabha if his five-year term
is up.
Second, when the leader of the majority -- also
known as the Treasury benches -- loses his
majority in the Lok Sabha. Then, the prerogative
passes to the President, who can ask another
leader to prove his majority in the Lok Sabha or
dissolve the Lok Sabha, and call for elections.
53. Ans: D
Exp: apart from the non-statutory instructions
issued by the government time to time, display of
national flag is governed by Emblems and Names
(prevention of improper use) act 1950, and
Prevention of Insults to National Honor act 1971,
Flag code of India 2002.
54. Ans: A
Exp: 53. Executive power of the Union
(1) The executive power of the Union shall be
vested in the President and shall be exercised by
him either directly or through officers subordinate
to him in accordance with this Constitution.
Article 75(3)- The Council of Ministers shall be
collectively responsible to the House of the People.
55. Ans: A
Exp:
Under Article 352, the President can
declare a national emergency when the security of
India or a part of it is threatened by war or
external aggression or armed rebellion. It may be
noted that the president can declare a national
emergency even before the actual occurrence of

war or external aggression or armed rebellion, if


he is satisfied that there is an imminent danger.
A proclamation of national emergency may be
applicable to the entire country or only a part of
it. The 42nd Amendment Act of 1976 enabled the
president to limit the operation of a National
Emergency to a specified part of India
56. Ans: D
Exp: VII schedule refers to division of legislative
subject between Centre and States : economic
and social planning.
Part IX refers to panchayats which are
responsible for local planning.
Sixth schedule- It was created to ensure that the
rights of tribals who are minorities within a state
or geographical area populated by a dominant
non-tribal population are not subsumed within
the rights framework of the latter. It gives them
the right to manage and redistribute their own
resources and plan for economic activities
through several structures.
57. Ans: B
Exp: The Vice President shall act as President in
the absence of the President due to death,
resignation, impeachment, or other situations.
The Vice President of India is also ex
officio Chairperson of the Rajya Sabha. He ceases
to perform as Chairman of Rajya Sabha while
acting as President.
Election of Vice President
Article 66(1) The Vice President shall be elected by
the members of an electoral college consisting of
the members of both Houses of Parliament in
accordance with the system of proportional
representation by means of the single transferable
vote and the voting at such election shall be by
secret ballot
58. Ans: D
Exp: The representatives of the States and of the
Union Territories in the Rajya Sabha are elected
by the method of indirect election. The
representatives of each State and two Union
territories are elected by the elected members of
the Legislative Assembly of that State and by the
members of the Electoral College for that Union
Territory, as the case may be, in accordance with
the system of proportional representation by
means of the single transferable vote. The
Electoral College for the National Capital Territory
of Delhi consists of the elected members of the
Legislative Assembly of Delhi, and that for

ANALOG IAS INSTITUTE The Right Choice of Achievers.


www.analogeducation.in

P a g e | 13
ias.analog@gmail.com

Puducherry consists of the elected members of


the Puducherry Legislative Assembly.
Rajya Sabha is a permanent House and is not
subject to dissolution. However, one-third
Members of Rajya Sabha retire after every second
year. A member who is elected for a full term
serves for a period of six years. The election held
to fill a vacancy arising otherwise than by
retirement of a member on the expiration of his
term of office is called Bye-election. A member
elected in a bye-election remains member for the
remainder of the term of the member who had
resigned or died or disqualified to be member of
the House under the Tenth Schedule.
59. Ans: A
Exp: In case of over-lapping of a matter between
the three Lists, predominance has been given to
the Union Legislature, as under the Government
of India Act, 1935. Thus, the power of the State
Legislature to legislate with respect to matters
enumerated in the State List has been made
subject to the power of the Union Parliament to
legislate in respect of matters enumerated in the
Union and Concurrent Lists, and the entries in
the State List have to be interpreted accordingly.
In the Concurrent sphere, in case of repugnancy
between a Union and a State law relating to the
same subject, the former prevails. If however, the
State law was reserved for the assent of the
President and has received such assent, the State
law
may
prevail
notwithstanding
such
repugnance. But it would still be competent for
Parliament to override such State law by
subsequent legislation.
60. Ans: B
Exp: The constitution gives the states power to
frame laws on all the subjects included in the
state list. But the central government has the
authority to interfere even in these powers of
states under special circumstances.
At the Resolution of Rajya Sabha
The Union Parliament will be authorized to pass a
law on a subject of state list about which the
Rajya Sabha passes a resolution with 2/3rds
majority of the members present and voting
saying that subject has attained national
importance. Such a resolution of the Rajya Sabha
will give the Parliament the Legislative power for
one year at a time. The power may be extended
yearly till the need is felt.
Request of the states

Art. 252 empower the Parliament to legislate on a


matter in the state list if two or more states desire
that any of the matter in the state list be
regulated by the Parliament. Any Act pressed
cannot be amended by a State Legislature to
which the law applies.
Failure of the constitutional machinery
The Parliament is empowered to pass laws on the
state subject for the state in which emergency
has been proclaimed because of the failure of
constitutional machinery. It may delegate his
Legislative Power concerning the states to the
President of India if it so deems necessary.
During emergency
The Parliament has the authority to pass laws on
any subjects of the state list during the
emergency proclaimed because of war and
external aggression etc. Such a law may be made
for the whole of India or any part thereof.
Enforcement of International treaties
The Parliament has the power to make law on
any item of the state in order to implement some
international treaty or agreement or convention.
Assent of President is essential
Some Bills are reserved by the Governor for the
signature of the President after being passed by
the state Legislature e.g. Bills concerning
restrictions as the powers of High Courts and
Bills concerning acquisitions on property by the
state by paying compensation etc. The President
has the power of absolute veto over the Bills
which are thus reserved by the Governors for his
signature.
A critical-examination-of legislative relation leaves
the impression that the centre is very powerful in
legislative matters and it can impose its will on
the state. All subjects of national importance are
in the Union list and in the concurrent list centre
is all powerful.
61. Ans: B
Exp:
Article 268A. Service tax levied by Union and
collected and appropriated by the Union and
the States.- (1) Taxes on services shall be levied
by the Government of India and such tax shall
be collected and appropriated by the Government
of India and the States in the manner provided in
clause (2).
(2) The proceeds in any financial year of any
such tax levied in accordance with the provisions
of clause (1) shall be

ANALOG IAS INSTITUTE The Right Choice of Achievers.


www.analogeducation.in

P a g e | 14
ias.analog@gmail.com

(a) collected by the Government of India and the


States;
(b) Appropriated by the Government of India and
the States, in accordance with such principles of
collection and appropriation as may be
formulated by Parliament by law.
62. Ans: B
Exp: The Interstate River Water Disputes Act,
1956 (IRWD Act) is an Act of the Parliament of
India enacted under Article 262 of Constitution of
India on the eve of reorganization of states on
linguistic basis to resolve the water disputes that
would arise in the use, control and distribution of
an interstate river or river valley. Article 262 of
the Indian Constitution provides a role for the
Central government in adjudicating conflicts
surrounding inter-state rivers that arise among
the state/regional governments. This Act further
has undergone amendments subsequently and
its most recent amendment took place in the year
2002.
River waters use / harnessing is included in
states jurisdiction (entry 17 of state list, Schedule
7 of Indian Constitution). However, union
government can make laws on regulation and
development of inter-State rivers and river valleys
when expedient in the public interest (entry 56
of union list, Schedule 7 of Indian Constitution).
When public interest is served, President may
also establish a interstate council as per Article
263 to inquire and recommend on the dispute
that has arisen between the states of India. IRWD
Act (section 2c2) validates the previous
agreements (if any) among the basin states to
harness water of an interstate river/ river valley.
"The SC indeed would have the the
jurisdiction to decide any dispute between states
in connection with the water supplies, if legal
rights and interests are concerned; but the
experience of many countries have shown that
the rule of law based on the analogy of private
proprietary interest in water do not afford
satisfactory basis for settling disputes between
states where interests of public at large in the
proper use of water supplies are involved "
63. Ans: B
Exp:
Article 148: Comptroller and AuditorGeneral of India
(1) There shall be a Comptroller and AuditorGeneral of India who shall be appointed by the

President by warrant under his hand and seal


and shall only be removed from office in like
manner and on the like grounds as a Judge of the
Supreme Court.
Article 151: Audit reports
(1) The reports of the Comptroller and AuditorGeneral of India relating to the accounts of the
Union shall be submitted to the President, who
shall cause them to be laid before each House
of Parliament.
(2) The reports of the Comptroller and AuditorGeneral of India relating to the accounts of a
State shall be submitted to the Governor of the
State, who shall cause them to be laid before
the Legislature of the State.
64. Ans: C
Exp: 323A. Administrative tribunals.Parliament may, by law, provide for the
adjudication or trial by administrative tribunals of
disputes and complaints with respect to
recruitment and conditions of service of persons
appointed to public services and posts in
connection with the affairs of the Union or of any
State or of any local or other authority within the
territory of India or under the control of the
Government of India or of any corporation owned
or controlled by the Government.
A law made under clause (1) may(a) Provide for the establishment of an
administrative tribunal for the Union and
a separate administrative tribunal for each
State or for two or more States;
(b) Specify the jurisdiction, powers (including
the power to punish for contempt) and
authority which may be exercised by each
of the said tribunals;
(c) Provide for the procedure (including
provisions as to limitation and rules of
evidence) to be followed by the said
tribunals;
(d) Exclude the jurisdiction of all courts, except
the jurisdiction of the Supreme Court and
high courts under article 136, with respect
to the disputes or complaints referred to in
clause (1);
(e) Provide for the transfer to each such
administrative tribunal of any cases
pending before any court or other
authority
immediately
before
the
establishment of such tribunal as would

ANALOG IAS INSTITUTE The Right Choice of Achievers.


www.analogeducation.in

P a g e | 15
ias.analog@gmail.com

have been within the jurisdiction of such


tribunal if the causes of action on which
such suits or proceedings are based had
arisen after such establishment;
(f) Repeal or amend any order made by the
President under clause (3) of article 371D;
(g) Contain such supplemental, incidental and
consequential
provisions
(including
provisions as to fees) as Parliament may
deem
necessary
for
the
effective
functioning of, and for the speedy disposal
of cases by, and the enforcement of the
orders of, such tribunals.
The provisions of this article shall have effect
notwithstanding anything in any other provision
of this Constitution or in any other law for the
time being in force.
65. Ans: B
Exp: The electoral roll is a list of all people in the
constituency who are register to vote in India
Elections. Only those people with their names on
the electoral roll are allowed to vote. The electoral
roll is revised every year to add the names of
those who are to turn 18 on 1st January of that
year or have moved into that constituency and to
remove the names of those who have died or
moved out.
66. Ans: A
Exp: Parliament exercise the control by asking
question to the ministers through its members,
by raising adjournment motions, cut motions,
censure motions or debates. More importantly the
Lok Sabha can pass a vote of no confidence
against the Council of Ministers which compels it
to resign collectively. Thus the parliament holds
the ministers responsible individually and
collectively. This critical function of the
Parliament ensures a responsive and responsible
government.
67. Ans: C
Exp: Legislative Council (Vidhan Parishad) of a
state comprises not more than one-third of total
number of members in legislative assembly of the
state and in no case less than 40 members
(Legislative Council of Jammu and Kashmir has
36 members vide Section 50 of the Constitution of
Jammu and Kashmir). About one-third of
members of the council are elected by members of
legislative assembly from amongst persons who
are not its members, one-third by electorates
consisting of members of municipalities, district

boards and other local authorities in the state,


one-twelfth by electorate consisting of persons
who have been, for at least three years, engaged
in teaching in educational institutions within the
state not lower in standard than secondary school
and a further one-twelfth by registered graduates
of more than three years standing. Remaining
members are nominated by Governor from among
those who have distinguished themselves in
literature, science, art, cooperative movement and
social service. Legislative councils are not subject
to dissolution but one-third of their members
retire every second year.
There is no provision of the joint session of both
the houses of legislature.
68. Ans: C
Exp: Article 102 - Disqualifications for
membership
(1) A person shall be disqualified for being chosen
as, and for being, a member of either House of
Parliament
(a) If he holds any office of profit under the
Government of India or the Government of any
State, other than an office declared by Parliament
by law not to disqualify its holder;
(b) If he is of unsound mind and stands so
declared by a competent court;
(c) If he is an undischarged insolvent;
(d) If he is not a citizen of India, or has voluntarily
acquired the citizenship of a foreign State, or is
under any acknowledgement of allegiance or
adherence to a foreign State;
(e) If he is so disqualified by or under any law
made by Parliament Explanation For the
purposes of this clause a person shall not be
deemed to hold an office of profit under the
Government of India or the Government of any
State by reason only that he is a Minister either
for the Union or for such State
(2) A person shall be disqualified for being a
member of either House of Parliament if he is so
disqualified under the Tenth Schedule
69. Ans: C
Exp: The PAC is formed every year with a
strength of not more than 22 members of which
15 are from Lok Sabha, the lower house of the
Parliament, and 7 from Rajya Sabha, the upper
house of the Parliament. The term of office of the
members is one year. The Chairman is appointed
by the Speaker of Lok Sabha. Its chief function is
to examine the audit report of Comptroller and

ANALOG IAS INSTITUTE The Right Choice of Achievers.


www.analogeducation.in

P a g e | 16
ias.analog@gmail.com

Auditor General (CAG) after it is laid in the


Parliament. CAG assists the committee during
the course of investigation. None of the 22
members shall be a minister in the government.
Estimates Committee suggested
alternative
policies in order to bring about efficiency and
economy in administration.
70. Ans: D
Exp: An amendment of this Constitution may be
initiated only by the introduction of a Bill for the
purpose in either House of Parliament, and when
the Bill is passed in each House by a majority of
the total membership of that House present and
voting, it shall be presented to the President who
shall give his assent to the Bill and thereupon the
Constitution shall stand amended in accordance
with the terms of the Bill.
If any question arises whether a Bill is a Money
Bill or not, the decision of the Speaker of the
House of the People thereon shall be final
The question of double jeopardy is not applicable
in the case of any departmental proceedings.
71. Ans: C
Exp: under chapter I (part V) of the Indian
Constitution only the President, Council of
Ministers, Attorny General are mentioned as part
of Union Executive. The CAG is mentioned under
Chapter V (Part V) as an independent body.
72. Ans: B
Exp:
Subject to the provisions of the
Constitution, the Legislative Assembly shall have
power to make laws for the whole or any part of
the National Capital Territory with respect to any
of the matters enumerated in the State of List or
in the Concurrent List in so far as any such
matter is applicable to Union territories except
matters with respect to Entries 1,2, and 18 of the
State List and Entries 44, 65 and 66 of that List
in so far as they relate to the said
Entries. Nothing in sub-clause (a) shall derogate
from the powers of Parliament under this
constitution to make laws with respect to any
matter for a Union Territory or any part thereof.
73. Ans: C
Exp: the sovereignty of people is a feature of a
democratic government but not a republic.
74. Ans: B
Exp: Right to equality before law (article 14) and
Right to Protection of Life and Personal Liberty
(article 21) are provided to aliens also.

75. Ans: A
Exp: a PIL can be produced before the court by
anyone if he is aggrieved himself or he stands for
someone else, who is aggrieved.
76. Ans: C
Exp: The commission presents its annual report
to the President not the Prime Minister. President
places the reports of the Commission before the
parliament.
77. Ans: A
Exp: Roles of the Prime Minister of India.
Forms the Government:
Allocation of Portfolios:
Shuffling of his Pack:
Chairman of the Cabinet:
Coordinator:
Leader of the Lok Sabha:
Link between the President and the Cabinet:
78. Ans: D
Exp: J & K has its own Constitution hence all the
given statements regarding it are correct.
79. Ans: B
Exp: The voting on demands for grants is an
exclusive privilege of the LokSabha-RajyaSabha
has no power on demands.
80. Ans: D
Exp: The salary and allowance of the Chief
Minister are determined by the State Legislature.
The Chief Minister acts as the chairman of the
concerned zonal council by rotation, holding office
for a period of one year at a time.
81. Ans: A
Exp: A political party is called a national party if
:- (i) it secures at least six percent(6%) of the valid
votes polled in any four or more states, at a
general election to the House of the People or, to
the State Legislative Assembly; and (ii) in
addition, it wins at least four seats in the House
of the People from any State or States. OR it wins
at least two percent (2%) seats in the House of the
People i.e. Lok Sabha, and these members are
elected from at least three different States.
82. Ans: A
Exp: When the Speaker is removed from office by
a resolution which is passed by a majority of all
the members of the House. While such a process
is underway, the Speaker cannot preside over the
House, but can take part in the proceedings of
the House.
83. Ans: D
Exp: All are true.

ANALOG IAS INSTITUTE The Right Choice of Achievers.


www.analogeducation.in

P a g e | 17
ias.analog@gmail.com

84. Ans: D
Exp: As the article 263 makes it clear, the InterState Council is not a permanent constitutional
body for coordination between the States of the
Union. It can be established 'at any time' if it
appears to the President that the public interests
would be served by the establishment of such a
Council.
The Council shall consist of:a)
Prime Minister Chairman
b)
Chief Ministers of all States -Member
c)
Chief Ministers of Union Territories having a
Legislative Assembly and Administrators of
UTs not having a Legislative Assembly
Member
d)
Six Union Ministers of Cabinet rank in the
Union Council of Ministers nominated by
the Prime Minister Member
The Presidential Order of 1990 has been
amended twice vide Orders dated 19 July 1990
and 24 December 1996 providing for Governor of
a State under President's rule to attend the
meeting of the Council and nomination by the
Chairman of permanent invitees from amongst
the other Union Ministers, respectively.
85. Ans: C
Exp: It is not mandatory for a member of a Rajya
Sabha be a domicile of the state from which
he/she is elected.
86. Ans: C
Exp: Legislative Procedure and Bicameralism
are borrowed from the constitution of UK,
whereas Federation with a strong Center are
borrowed from the constitution of Canada.
87. Ans: B
Exp: the directives are not justiciable in the
Court of Law.
88. Ans: B
Exp: Once approved it may continue for indefinite
period.
89. Ans: C
Exp:
Article 58 of the Constitution sets the
principle qualifications one must meet to be
eligible to the office of the President. A President
must be:

A citizen of India

Of 35 years of age or above

Qualified to become a member of the Lok


Sabha

A person shall not be eligible for election as


President if he holds any office of profit under
the Government of India or the Government of
any State or under any local or other authority
subject to the control of any of the said
Governments.
90. Ans: C
Exp: Article 363 in The Constitution Of India
1949
363. Bar to interference by courts in disputes
arising out of certain treaties, agreements, etc
(1) Notwithstanding anything in this Constitution
but subject to the provisions of Article 143,
neither the Supreme Court nor any other court
shall have jurisdiction in any dispute arising out
of any provision of a treaty, agreement, covenant,
engagement, sanad or other similar instrument
which was entered into or executed before the
commencement of this Constitution by any Ruler
of an Indian State and to which the Government
was a party and which has or has been continued
in operation after such commencement, or in any
dispute in respect of any right accruing under or
any liability or obligation arising out of any of the
provisions of this Constitution relating to any
such treaty, agreement, covenant, engagement,
sanad or other similar instrument.
91. Ans: A
Exp: Chief Minister is the chief spokesman of the
state government.
92. Ans: D
Exp: All are correct.
93. Ans: C
Exp: The Chairman and members hold office for
a term of four years or until they attain the age
65 years whichever is earlier.
94.Ans: D
Exp: all are true.
95. Ans: A
Exp: article 124 (4) A Judge of the Supreme
Court shall not be removed from his office except
by an order of the President passed after an
address by each House of Parliament supported
by a majority of the total membership of that
House and by a majority of not less than two
thirds of the members of that House present and
voting has been presented to the President in the
same session for such removal on the ground of
proved misbehaviour or incapacity.
For high court judges, Article 217 (b) says
his/her removal from the office is to be done in

ANALOG IAS INSTITUTE The Right Choice of Achievers.


www.analogeducation.in

P a g e | 18
ias.analog@gmail.com

the same manner provided in Article 124 (4) cited


above.
The speaker/ Chairman may or may not admit
the motion.
96. Ans: D
Exp: The Public Accounts Committee (PAC) is a
committee of selected members of Parliament,
constituted by the Parliament of India, for the
auditing of the revenue and the expenditure of
the Government of India.
The PAC is formed every year with a
strength of not more than 22 members of which
15 are from Lok Sabha, the lower house of the
Parliament, and 7 from Rajya Sabha, the upper
house of the Parliament. The term of office of the
members is one year. The Chairman is appointed
by the Speaker of Lok Sabha. Since 1967, the
chairman of the committee is selected from the
opposition. Earlier, it was headed by a member of
the ruling party. Its chief function is to examine
the audit report of Comptroller and Auditor
General (CAG) after it is laid in the Parliament.
CAG assists the committee during the course of
investigation. None of the 22 members shall be a
minister in the government.
Its recommendations are not binding.
97. Ans: A
Exp: There is no provision in the Constitution
that every child below ten years of age whose
family is living below the poverty line is provided
free food by the state.

98. Ans: A
Exp: CAT is guided by the principles of natural
justice.
99. Ans: D
Exp: all are true.
100. Ans: A
Exp: Inner Line Permit (ILP) is an official travel
document issued by the Government of India to
allow inward travel of an Indian citizen into a
protected area for a limited period. It is obligatory
for Indian citizens from outside those states to
obtain a permit for entering into the protected
state. The document is an effort by the
government to regulate movement to certain
areas located near the international border of
India. This is an offshoot of the Bengal Eastern
Frontier Regulations, 1873, which protected
Crown's interest in the tea, oil and elephant trade
by prohibiting "British subjects" from entering
into these "Protected Areas" (to prevent them from
establishing any commercial venture that could
rival the Crown's agents) . The word "British
subjects" was replaced by Citizen of India in
1950. Despite the fact that the ILP was originally
created by the British to safeguard their
commercial interests, it continues to be used in
India, officially to protect tribal cultures in
northeastern India.
The states which require the permit are
Arunachal Pradesh, Mizoram, Nagaland and
Sikkim.

*****

ANALOG IAS INSTITUTE The Right Choice of Achievers.


www.analogeducation.in

P a g e | 19
ias.analog@gmail.com

Potrebbero piacerti anche